You are on page 1of 55

Point & Straight Line

IIT-JEE Syllabus
1. Coordinate systems
2. Distance formula & its application
3. Section formula
4. Area of triangle & Area of Quadrilateral
5. Centers of triangle
6. Locus, Transformation of axes.
7. Equation of straight line in different form (upto parametric form)
8. Angle between lines
9. Equation of line Parallel & perpendicular to a line
10.
11.

Equation of angle bisectors

Equation of line making definite angle to a given line


12.

Family of lines

Total No. of questions in Point & Straight line are:


Solved examples......61
Exercise # 1 ..54
Exercise # 2 ..52
Exercise # 3 ..37
Exercise # 4 ..19
Exercise # 5 ..31
Total No. of questions..254

*** Students are advised to solve the questions of exercises in the same sequence or as directed by the
faculty members.

Index : Preparing your own list of Important/Difficult Questions


Instruction to fill
(A) Write down the Question Number you are unable to solve in column A below, by Pen.
(B) After discussing the Questions written in column A with faculties, strike off them in the manner
so that you can see at the time of Revision also, to solve these questions again.
(C) Write down the Question Number you feel are important or good in the column B.

EXERCIS
E NO.

COLUMN :A

COLUMN :B

Questions i am unable
to solve in first attempt

Good/Important questions

Advantages
1. It is advised to the students that they should prepare a question bank for the revision as it is very
difficult to solve all the questions at the time of revision.
2. Using above index you can prepare and maintain the questions for your revision.

KEY CONCEPTS
(a) Two polygons are similar if (i) their
corresponding angles are equal, (ii) the length
of their corresponding sides are proportional.
(Both condition are independent & necessary)
In case of a triangle, any one of the condition
is sufficient, other satisfies automatically.
(b) Thales
Theorem
(Basic
Proportionality
Theorem): In a triangle,
a line drawn parallel
to one side, to intersect the
other sides in distinct
points, divides the two
sides in the same ratio.
Converse : If a line divides any two sides of
a triangle in the same ratio then the line must
be parallel to the third side.
(c) Similarity Theorem :
(i) AAA similarity : If in two triangles,
corresponding angles are equal i.e. two
triangles are equiangular, then the triangles are
similar.
(ii) SSS similarity : If the corresponding sides
of two triangles are proportional, then they
are similar.
(iii) SAS similarity : If in two triangles,
one pair of corresponding sides are
proportional and the included angles are
equal then the two triangles are similar.
(iv) If two triangles are similar then
(1) They are equiangular
(2) The ratio of the corresponding
(I) Sides (all), (II) Perimeters,
(III) Medians, (IV) Angle bisector
segments, (V) Altitudes are same
(converse also true )
(3) The ratio of the areas is equal to the
ratio of the squares of corresponding
(I) Sides (all), (II) Perimeters,
(III) Medians, (IV) Angle bisector
segments, (V) Altitudes (converse also
true)
(d) Pythagoras Theorem :
(i) In a right triangle the square of hypotenuse
is equal to the sum of square of the other two
sides.
Converse : In a triangle if square of one side
is equal to sum of the squares of the other
two sides, then the angle opposite to the first
side is a right angle.

(ii) In obtuse

AC2 = AB2 + BC2 + 2BC . BD

(iii) In Acute

AC2 = AB2 + BC2 2BC . BD


(e) The internal/external bisector of an angle of a
triangle divides the opposite side internally/
externally in the ratio of sides containing the
angle (converse is also true)
(f) The line joining the mid points of two sides
of a is parallel & half of the third side. (It's
converse also true)
(g) (i) The diagonals of a trapezium divided each
other proportionally. (converse is also true)
(ii) Any line parallel to the parallel sides of a
trapezium divides the non parallel sides
proportionally.
(iii) If three or more parallel lines are
intersected by two transversals, then
intercepts made by them on transversals
are proportional.
(h) In any triangle the sum of squares of any two
sides is equal to twice the square of half of
the third side together with twice the square
of the median which bisects the third side.
(i) In any triangle the three times the sum of
squares of the sides of a triangle is equal to
four times the sum of the square of the
medians of the triangle.
(j) The altitudes, medians and angle bisectors of
a triangle are concurrent among themselves.

Coordinate geometry is a combination of algebra and


geometry. A systematic study of geometry by the use
of algebra was first carried out by celebrated French
philosopher and mathematician Rene Descartes. The
resulting combination of analysis and geometry is
referred as analytical geometry.

51

In two dimensional coordinate system, two lines are


used; the lines are at right angles, forming a
rectangular coordinate system. The horizontal axis is
the x-axis and the vertical axis is y-axis. The point of
intersection O is the origin of the coordinate system.
Distances along the x-axis to the right of the origin
are taken as positive, distances to the left as negative.
Distances along the y-axis above the origin are
positive : distances below are negative.

If A(x1, y1) and B(x2, y2) are two points, then


AB = ( x 2 x 1 ) 2 ( y 2 y1 ) 2
Note :
(a) Three given points A, B and C are collinear,
when sum of any two distances out of AB,
BC, CA is equal to the remaining third
otherwise the points will be the vertices of
triangle.
(b) Let A, B, C & D be the four given points in a
plane. Then the quadrilateral will be :
(i) Square if AB = BC = CD = DA &
AC = BD ; AC BD
(ii) Rhombus if AB = BC = CD = DA &
AC BD ; AC BD

The position of a point anywhere in the plane can be


specified by two numbers, the coordinates of the
point, written as (x, y). The x-coordinate
(or abscissa) is the distance of the point from the yaxis in a direction parallel to the x-axis
(i.e. horizontally). They y-coordinate (or ordinate) is
the distance from the x-axis in a direction parallel to
the y-axis (vertically). The origin O is the point (0,
0).

A coordinate system in which the position of a point


is determined by the length of a line segment from
fixed origin together with the angle or angles that the
line segment makes with a fixed line or lines. The
origin is called the pole and the line segment is the
radius vector (r).
The angle between the polar axis and the radius
vector is called the vectorial angle. By convention,
positive values of are measured in an anticlockwise
sense, negative values in clockwise sense. The
coordinates of the point are then specified as (r, ).

(iii) Parallelogram if AB = DC, BC = AD;


AC BD ; AC BD
(iv) Rectangle if AB = CD, BC = DA,
AC = BD ; AC BD
Quadrilateral

Diagonals

(i) Parallelogram

Not equal

(ii) Rectangle

Equal

(iii) Rhombus

Not equal

(iv) Square

Equal

Angle
between
diagonals

Note :
(i) Diagonal of square, rhombus, rectangle
and parallelogram always bisect each other.
(ii) Diagonal of rhombus and square bisect
each other at right angle.
5.1 Position of three points :

If (x, y) are Cartesian co-ordinates of a point P, then :


x 2 y2 ,
x = r cos, y = r sin and r =
y

= tan1
x

Let A, B, C are points lie in a plane then two


condition arises.
5.1.1 Collinearity of three given points :
The three given points A, B, C are collinear
i.e., lie on the same straight line, if

51

(i) any of the three points (say B) lie on the


straight line joining the other two points.

points P(x1, y1) and Q(x2, y2) in the


of : 1 is given by

Note :

x 1 x 2 y 1 y 2
,

, ( 1) (Imp.)
1
1

The other conditions for collinearity is


(i) area of ABC is zero

x1
It means = x 2
x3

y1
y2

1
1

y3

(ii) Lines formed by joining (x1, y1) and (x2, y2)


is divided by
(a) X-axis in the ratio y1/y2

=0

(b) Y-axis in the ratio x1/x2


If ratio is positive the axis divide it
internally and if negative divides
externally.

[x1 (y2 y3) + x2 (y3 y1) + x3 (y1 y2)] = 0


(ii) Slope of AB = slope of BC
Slope Formula

(iii) Line Ax + By + C = 0 divides the line


joining the points (x1, y1) and (x2, y2) in
the ratio : 1, then

If is the anticlockwise angle at which a


straight line is inclined to the positive direction
of the x-axis, and 0 < 180, 90, then
the slope of the line, denoted by m, is defined
by m = tan .
Note :
(i) If is 90, m does not exist, but the line
is parallel to the y-axis.

Ax By C

1
1

=
Ax 2 By 2 C

If is positive it divides internally if is


negative then externally.
(iv) If P divides AB internally in the ratio m : n
& Q divides AB externally in the ratio
m : n then P & Q are said to be harmonic
conjugate of each other w.r.t. AB.
1
2
1
=
+
AQ
AB
AP

(ii) If = 0, then m = 0 and the line is parallel to


the x-axis.

Mathematically,

(iii) If A(x1, y1) and B(x2, y2), x1 x2, are points on


a straight line, then the slope m of the line is
given by: m = (y1 y2)/(x1 x2)

i.e. AP, AB & AQ are in H.P.

ratio

We see later these formula

Let A(x1, y1), B(x2, y2) and C(x3, y3) are vertices of
any triangle ABC, then
7.1 Centroid :

Coordinates of a point which divides the line


segment joining two points P (x1, y1) and Q (x2, y2) in
the ratio m1 : m2 are

m1 x 2 m 2 x 1 m 1 y 2 m 2 y1

,
m1 m 2
m1 m 2

(i)

The centroid is the point of intersection of the


medians (line joining the mid point of sides
and opposite vertices) centroid divides the
median in the ratio of 2 : 1. Coordinates of

x 1 x 2 x 3 y1 y 2 y 3
,

3
3

(internal division)

m 1 x 2 m 2 x 1 m1 y 2 m 2 y1

,
m1 m 2
m 1 m 2

(ii)

(external division)
When m1, m2 are of opposite sign, the division is
external.
Note :
(i) Co-ordinates of any point on the line
segment which divide the line joining two

7.2 Incentre :

51

The incentre of the triangle is the point of


intersection of internal bisector of the angle.
Also it is a centre of a circle touching all the
sides of a triangle. Co-ordinates of incentre

ax1 bx 2 cx 3 ay1 by 2 cy 3
,

abc
abc

If a triangle is right angle , then its circumcentre is


the mid point of hypotenuse.
For Example : in a right angle triangle ABC the
point D is known as the circumcentre of a
triangle

where a, b, c are the sides of triangle ABC

7.4 Orthocentre :
It is the point of intersection of perpendicular
drawn from vertices on opposite sides called
altitudes of a triangle and can be obtained by
solving the equation of any two altitudes.
Note :
(i) Angle bisector divides the opposite sides
in the ratio of remaining sides
eg.

BD
AB
c
=
=
AC
DC
b
about orthocentre of we will be discuss later.
Note :
If a triangle is right angle triangle, then ortho
centre is the point where right angle is formed.

(ii) Incentre divides the angle bisector in the


ratio
(b + c) : a, (c + a) : b and (a + b) : c
7.3 Circumcentre :
It is the point of intersection of perpendicular
bisectors of the sides of the triangle. It is also
the centre of a circle passing through the
vertices of the triangle. If O is the circumcentre
of any ABC.
2

Then OA = OB = OC

Remarks :
(i) If the triangle is equilateral, then centroid,
incentre, orthocentre, circumcentre coincides.
(ii) Orthocentre, centroid and circumcentre are
always collinear and centroid divides the line
joining orthocentre and circumcentre in the
ratio 2 : 1.
(iii) In an isosceles triangle centroid, orthocentre,
incentre, circumcentre lies on the same line.
7.5 Ex-centres :
The centre of the circle which touches side
BC and the extended portions of sides AB and
AC is called the ex-centre of ABC with
respect to the vertex A. It is denoted by I 1 and
its coordinates are

ax 1 bx 2 cx 3 ay 1 by 2 cy 3
,

abc
abc

I1 =

Note:

51

I.

Assumes the coordinate of the point say


(h, k) whose locus is to be find.

II. Write the given condition involving (h, k)


III. Eliminate the variable(s) if any
Similarly ex-centers of ABC with respect to
vertices B and C are denoted by I 2 and I3
respectively, and

ax 1 bx 2 cx 3 ay1 by 2 cy 3
,
,
abc
a bc

IV. Replace h

x and k

The equation so obtained is the locus of


the point which moves under some definite
condition.

I2 =

ax 1 bx 2 cx 3 ay1 by 2 cy 3
,

abc
abc

I3 =

Let A(x1, y1), B(x2, y2) and C(x3, y3) are vertices of a
triangle, then
x1
1
x2
Area of ABC =
2
x3

y1
y2

1
1

y3

Let (x, y) be any point then its image with


respect to
(i) x axis

(x, y)

(ii) y axis

(x, y)

(iii) origin

(x, y)

(iv) Line y = x

1
|[x1(y2 y3) + x2(y3 y1) + x3(y1 y2)]|
2
To remember the above formula, take the help of the
following method :
=

(a) Reflection (Image) of a point :

1
2

= | 1 [(x1y2 x2y1) + (x2y3 x3y2) + (x3y1 x1y3)]|


2
Remarks :
(i) If the area of triangle joining three points
is zero, then the points are collinear.
(ii) Area of Equilateral triangle
If altitude of any equilateral triangle is p,
p2
then its area =
. If 'a' be the side of
3

(y, x)

(b) A triangle is isosceles if any two of its


median are equal.
(c) Triangle having integral coordinate can
be equilateral.

never

(d) If arx + bry + cr = 0 (r = 1, 2, 3) are the


sides
of a triangle then the area of the
triangle
is
given by

1
2C1 C 2 C 3

a1

b1

c1

a2
a3

b2
b3

c2
c3

where C1, C2, C3 are the cofactor of c1, c2, c3


in the determinant.
(e) Area of parallelogram
(i) Whose sides are a and b and angle
between them is is given by = ab sin

a2 3

equilateral triangle, then its area =

It is the path or the curve traced by a moving point


satisfying the given condition.
9.1 Equation to the locus of a point :
The equation to the locus of a point is the
algebaric relation which is satisfied by the
coordinates of every point on the locus of the
point.

(ii) Whose length of perpendicular from one


vertices to the opposite sides are p 1 and
p2 and angle between sides is is given
by =

p1p 2
sin

Step taken to find the equation of locus

51

(f) A triangle having vertices ( at 12 , 2at1), ( at 22 , 2at2)


and ( at 32 , 2at3) then
area of = a2[(t1 t2) (t2 t3) (t3 t1)]
(g) Area of formed by coordinate axis and the
c2
line ax + by + c is
2ab

Slope of a line means the inclination of the line with


the positive direction of x-axis . Thus the
trigonometrical tangent of the angle that a line makes
with the positive direction of the x-axis in
anticlockwise sense is called slope or gradient of a
line denoted by (m). Let the line AB make angle
with the x-axis then Gradient of line
AB = m = tan

(h) Area of rhombus formed by


|ax| + |by| + c = 0 is

2c 2
ab

(i) Three points A(x1, y1), B(x2, y2), C(x3, y3) are
collinear if slope of AB = slope of BC.

y3 y 2
y 2 y1
=
x 2 x1
x3 x2

(j) To remove the term of xy in the equation


ax2 + 2hxy + by2 = 0. The angle through which
the axis must be turned
=

1
2h
.
tan1
2
a b

A straight line is a curve such that every point on the


line segment joining any two points on it lie on it.

Note :
(i) Slope of x-axis or a line parallel to x-axis
is Zero.
(ii) Slope of y-axis or a line parallel to y-axis
is m = tan 90 =
14.1 Slope of a line in term of coordinates of
any two point on it :
Let P (x1, y1) & Q (x2, y2) be two point on a
line making an angle with positive direction
of x-axis.

A relation between x & y which is satisfied by


coordinates of every point lying on a line is called
the equation of straight line. Every first degree
equation in x, y i.e. ax + by + c = 0 represents a line.
Thus a line is also defined as the locus of a point
satisfying the condition ax + by + c = 0 where
a, b, c are constant.
In PQN, tan =
(i) Equation of x-axis y = 0

QN y 2 y1

PN x 2 x 1

y 2 y1
x 2 x1

(ii) Equation of a line parallel to x-axes at a distance


of b y = b

m = tan =

(iii) Equation y-axis x = 0

Thus slope of a line is

(iv) Equation of a line parallel to y-axis and at a


distance of a x = a

m = tan =

y 2 y 1 Diff . of ordinate

x 2 x 1 Diff . of abscissa

Note:
The combined equation of the coordinate
axis is xy = 0
15.1 Slope Intercept form :
The equation of a line with slope m and
making an intercept c on y-axis is y = mx + c

51

If the line passes through the origin, then c = 0.


Thus the equation of a line with slope m and
passing through the origin y = mx.
15.2 Two points form equation of line :
Let m be the slope of the line passing
through (x1, y1) & (x2, y2). Then,
m=

y 2 y1
x 2 x1

So the equation of the line is


y y1 = m (x x1)

...(1)

putting the value of m in eq. (1)

x
y
+
=1
p sec
p cos ec

x cos + y sin = p (p > 0) and [0, 2]


which is the required equation of line
15.5 Parametric or distance form of a line :

(y y1) =

y 2 y1
(x x1)
x 2 x1

This is the required equation of the line in two


points.
15.3 Intercept form of a line :

Let the given line meets x-axis at A and y-axis


at B and passes through the point Q(x1, y1).
Let P(x, y) be any point on the line at a
distance r from Q(x1, y1). Then
QN = ML = OL OM = x x1
and PN = PL NL = y y1

x x1
r

Let AB be the line which cuts off intercepts


OA = a and OB = b on the x and y-axis and
Let P(x, y) be any point on the line

cos=

Area of OAB = Area of OPA + Area of OPB

Again sin =

QN x x 1

PQ
r

From PQN, cos =

PN
PQ

...(i)
sin =

y y1
...(ii)
r

x y

= 1(a 0 , b 0)
a b

This is the equation of line in the intercept


form.
15.4 Normal (Perpendicular) form of a line :
Let the line AB be such that the length of the
perpendicular OQ from the origin O to the
line be P and XOQ = .

From (i) & (ii) we get

x x 1 y y1

r
cos
sin

by using intercept form of line we get

51

(a) If be the angle between two lines : y = m 1x + c1

This is the equation of the line in the distance


form.

m m

1
2

and y = m2x + c2, then tan =


1 m1m 2

Note:
(i) The equation of the line is

Note :

x x1 y y1

= r where r [0, )
cos
sin
x x1 = r cos and y y1 = r sin
x = x1 r cos and y = y1 r sin
Thus the coordinates of any point on the
line at a distance r from the given point
(x1, y1) are (x1 r cos, y1 r sin).

(ii) If is variable and x1, y1 and r are fixed


then the same equation will represent a
circle whose centre is (x1, y1) and radius
is r.

(i) There are two angles formed between two


lines but usually the acute angle is taken as
the angle between the lines. So we shall find
from the above formula only by taking
positive value of tan.
(ii) Let m1, m2, m3 are the slopes of three lines
L1 = 0; L2 = 0; L3 = 0 where
m1 > m2 > m3 then the interior angles of the
ABC found by these lines are given by,

m2 m3
m1 m 2
; tanB =
&
1 m1 m 2
1 m2m3
m 3 m1
tanC =
1 m 3 m1
tanA =

(iii) This form is selected where the portion of


a line or line segment is to be discussed.
15.6 Determinant form equation of line :
Let P(x1, y1), Q(x2, y2) are the two given point
then equation of the line passing through these

y 2 y1
is given by (y y1) =
(x x1)
x 2 x1

This can also be written in another form.


x
y 1
x 1 y1 1 0
x 2 y2 1

of

this

line

a
b

If two lines of slopes m1 and m2 are


perpendicular, then the angle between them
is of 90
tan90 =

15.8 General form : We know that a first degree


equation in x and y, ax + by + c = 0 always
represents a straight line. This form is known
as general form of straight line.
(i) Slope

(b) Condition for perpendicular lines:

Thus when two lines are perpendicular the


product of their slopes is 1. If m is the slope
of a line then the slope of a line perpendicular
1
to it is
.
m
Note:
(i) Lines a1x + b1y + c1 = 0 & a2x + b2y + c2 = 0
are parallel, if

coeff . of x
coeff . of y

(ii) Intercept by this line on x-axis =


intercept by this line on y-axis =

c
and
a

c
b

(iii) To change the general form of a line to


normal form, first take c to right hand side
and make it positive, then divide the whole
equation by a 2 b 2 .

m 2 m1
m1m2 = 1
1 m 2 m1

a1 a 2

b1 b 2
(ii) Lines a1x + b1y + c1= 0 & a2x + b2y + c2 = 0
are perpendicular, if a1a2 + b1b2 = 0
(c) Condition for Coincident lines :
Two lines a1x + b1y + c1 = 0 & a2x + b2y + c2 = 0
are coincident only and only if.

a1 b1 c1

.
a 2 b2 c2

51

(a) Equation of line parallel to line ax + by + c = 0 is


ax + by + = 0
(b) Equation of line perpendicular to
ax + by + c = 0 is bx ay + k = 0

line

Here , k are parameters and their values are


obtained with the help of additional information
given in the problem.
Note:
To write a line perpendicular to a given line
following steps are taken
Step I : Interchange x and y.
Step II : If the coefficients of x and y in the
given equation are of the same sign
make them of opposite signs and if the
coefficients are of opposite signs
make them of the same sign.
Step III : Replace the given constant by a new
constant which is determined by a given
condition.

Equations of lines passing through a point (x 1, y1)


and making an angle , with the line y = mx + c is
written as :
y y1 =

m tan
(x x1)
1 m tan

The coefficients of x & y in both equations


should be same.
(b) The area of the parallelogram =

p1p 2
, where
sin

p1 & p2 are distances between two pairs of


opposite sides & is the angle between any
two adjacent sides. Note that area of the
parallelogram bounded by the lines y = m1x + c1,
y = m1x + c2 and y = m2x + d1, y = m2x + d2
(c 1 c 2 )(d 1 d 2 )
is given by
.
m1 m 2
To find the distance between these two lines
following steps are taken
Step I : Choose a point on any one of the line by
giving a particular value of x or y.
Step II : Then find the length of the perpendicular
from the point to line. The length so obtained is the
required distance between the parallel lines.

Let the given line be ax + by + c = 0 and P(x 1, y1),


Q(x2, y2) be two points. If the quantities ax 1 + by1 + c
and ax2 + by2 + c have the same signs, then both the
points P and Q lie on the same side of the line ax +
by + c = 0. If the quantities ax1 + by1 + c and ax2 +
by2 + c have opposite signs, then they lie on the
opposite sides of the line.

Let the equations of two lines be


Length of perpendicular from a point (x 1, y1) on the
line ax + by + c = 0 is p =

ax 1 by 1 c
a 2 b2

In particular the length of the perpendicular from the


origin on the line ax + by + c = 0 is
|c|
p=
2
a b2

(a) The distance between two parallel lines


ax + by + c1 = 0 and ax + by + c 2 = 0 is
| c1 c 2 |
=
a 2 b2
Note :

a1x + b1y + c1 = 0

...(i)

and a2x + b2y + c2 = 0

...(ii)

Let these two lines intersect at a point P (x1, y1). Then


(x1, y1) satisfies each of the given equations
a1x1 + b1y1 + c1= 0

and

a2x1 + b2y1 + c2 = 0
Solving these two equations

y1
x
1

b 1 c 2 b 2 c 1 c1 a 2 c 2 a 1 a 1 b 2 a 2 b 1
Hence the coordinates of the point of intersection of
(i) and (ii) are

b 1 c 2 b 2 c1 c 1 a 2 c 2 a 1

,
a 1 b 2 a 2 b 1 a 1 b 2 a 2 b1

51

(a) Three lines a1x + b1y + c1 = 0; a2x + b2y + c2 = 0


and a3x + b3y + c3 = 0 are concurrent, if

a1
a2

b1
b2

c1
c2

a3

b3

c3

=0

(b) To test the concurrency of three lines, first find


out the point of intersection of any two of the
three lines. If this point lies on the remaining line
(i.e. coordinates of the point satisfy the equation
of the line) then the three lines are concurrent
otherwise not concurrent.

Let P(x, y) be any point, then its image with respect


to

then x = x' + , y = y' +


or

x' = x , y' = y

Thus if origin is shifted to point (, )


without rotation of axes, then new equation of
curve can be obtained by putting x + in place
of x and y + in place of y.
(b) Rotation of axes without shifting the origin :
Let O be the origin. Let P (x, y) with
respect to axes OX and OY and let P (x', y')
with respect to axes OX' and OY' where
X'OX = YOY' =

(a) x-axis is Q(x, y)


(b) y-axis is R(x, y)
(c) origin is S(x, y)
(d) line y = x is T(y, x)

(a) Shifting of origin without rotation of axes :


Let P (x, y) with respect to axes OX and OY.
Let O' (, ) is origin with respect to axes
OX and OY and let P (x', y') with respect to
axes O'X' and O'Y', where OX and O'X' are
parallel and OY and O'Y' are parallel.

then x = x' cos y' sin


y = x' sin + y' cos
and x' = x cos + y sin
y' = x sin + y cos
The above relation between (x, y) and (x', y')
can be easily obtained with the help of
following table
x

x'

cos

sin

y'

sin

cos

Old
New

51

not containing the origin. Rewrite the


equations, ax + by + c = 0 & a'x + b'y + c' = 0
such that the constant terms c, c' are positive.
Then;
The equation of a family of lines passing through the
point of intersection of a1x + b1y + c1 = 0 & a2x + b2y
+ c2 = 0 is given by (a 1x + b1y + c1) + k(a2x + b2y +
c2) = 0,where k is an arbitrary real number.
Note :
If u1 = ax + by + c, u 2 = a'x + b'y + d,
u3 = ax + by + c', u4 = a'x + b'y + d' then,
u1 = 0; u2 = 0; u3 = 03; u4 = 0 form a
parallelogram.
u2u3 u1u4 = 0 represents the diagonal BD.
Proof :
Since it is the first degree equation in x & y it
is a straight line. Secondly point B satisfies
the equation because the co-ordinates of B
satisfy u2 = 0 and u1 = 0.
Similarly for the point D. Hence the result.
On the similar lines u1u2 u3u4 = 0 represents
the diagonal AC.
Note :
The diagonal AC is also given by u 1 + u4 = 0
and u2 + u3 = 0, if the two equations are
identical for some and .
[For getting the values of & compare the
coefficients of x, y & the constant terms].

ax by c
2

(ii) To discriminate between the acute angle


bisector & the obtuse angle bisector
If be the angle between one of the lines &
one of the bisectors, find tan.
If |tan | < 1, then 2 < 90 so that this
bisector is the acute angle bisector.

= +

(iv) To discriminate between acute angle bisector


& obtuse angle bisector proceed as follows
write ax + by + c = 0 & a'x + b'y + c' = 0 such
that constant terms are positive. If aa' + bb' < 0,
then the angle between the lines that contains
the origin is acute and the equation of the
bisector of this acute angle is
ax by c
2

a b

ax by c
a 2 b2

a ' x b ' y c'

=+

a ' 2 b' 2

a ' x b ' y c'


a ' 2 b' 2

therefore

is the equation

of other bisector. If, however, aa' + bb' > 0,


then the angle between the lines that contains
the origin is obtuse & the equation of the
bisector of this obtuse angle is
ax by c
a 2 b2
ax by c
a 2 b2
equation

(i) Equations of the bisectors of angles between


the lines ax + by + c = 0 & a'x + b'y + c' = 0
ax by c
a ' x b ' y c'
(ab' a'b) are :
=
2
2
a b
a ' 2 b' 2

a ' x b ' y c'

given the
a b
a ' 2 b' 2
equation of the bisector of the angle containing
a ' x b ' y c'
ax by c
the origin &
=
2
2
a b
a ' 2 b' 2
gives the equation of the bisector of the angle
not containing the origin.
2

= +
=

a ' x b ' y c'


a ' 2 b' 2
a ' x b ' y c'

; therefore

a ' 2 b' 2
of other bisector.

is

the

(v) Another way of identifying an acute and obtuse


angle bisector is as follows :
Let L1 = 0 & L2 = 0 are the given lines & u 1 = 0
and u2 = 0 are the bisectors between L 1 = 0 &
L2 = 0. Take a point P on any one of the lines
L1 = 0 or L2 = 0 and drop perpendicular on
u1 = 0 & u2 = 0 as shown. If,
|p| < |q| u1 is the acute angle bisector.
|p| > |q| u1 is the obtuse angle bisector.
|p| = |q| the lines L1 & L2 are perpendicular.

If |tan | > 1, then we get the bisector to be


the obtuse angle bisector.
(iii) To discriminate between the bisector of the
angle containing the origin & that of the angle

51

(i) ax2 + 2hxy + by2 + 2gx + 2fy + c = 0 represents


a pair of straight lines if
abc + 2fgh af 2 bg2 ch2 = 0,
a

i.e. if h
g

b
f

f =0
c

(ii) The angle between the two lines representing


by a general equation is the same as that
between the two lines represented by its
homogeneous part only.

Note :
Equation of straight lines passing, through
P(x1, y1) & equally inclined with the lines
a1x + b1y + c1 = 0 & a2x + b2y + c2 = 0 are those
which are parallel to the bisectors
between these
two lines & passing through the point P.

(iii) If be the angle between the lines, then


2
tan = 2 h ab
ab

Obviously these lines are


(a) Parallel, if = 0, h2 = ab or if h2 = ab and
bg2 = af 2

(i) A homogeneous equation of degree two of the


type ax2 + 2hxy + by2 = 0 always represents a
pair of straight lines passing through the origin &
if :
(a) h2 > ab lines are real & distinct.
(b) h = ab lines are coincident
2

(c) h2 < ab lines are imaginary with real


point of intersection i.e. (0, 0)
(ii) If y = m1x & y = m2x be
represented by ax2 + 2hxy
2h
m1 + m2 =
& m1m2 =
b

the two equations


+ by2 = 0, then
a
.
b

(iii) If is the acute angle between the pair of straight


lines represented by, ax2 + 2hxy + by2 = 0, then; tan
=

h ab
a b

The condition that these lines are :


(a) At right angles to each other is a + b = 0.
i.e. coefficient of x2 + coefficient of y2 = 0.
(b) Coincident is h2 = ab.
(c) Equally inclined to the axis of x is h = 0
i.e. coeff. of xy = 0.
Note :
A homogeneous equation of degree n represents n
straight lines passing through origin.

(b) Perpendicular, if a + b = 0 i.e.


coeff. of x2 + coeff. of y2 = 0
Note :
(a) The joint equation of a pair of straight lines
joining origin to the points of intersection of
the line given by
x + my + n = 0

.(i)

& the 2nd degree curve:


ax2 + 2hxy + by2 + 2gx + 2fy + c = 0 (ii)

x my

is ax2 + 2hxy + by2 + 2gx

x my
x my
+c

n
n

2fy

= 0 ...(iii)

(iii) is obtained by homogenizing (ii) with the


help of (i), by writing (i) in the form :
x my

=1
n

(b) The equation to the straight lines bisecting the


angle between the straight lines,
ax2 + 2hxy + by2 = 0 is

xy
x 2 y2
=
.
h
ab

(c) The product of the perpendiculars, dropped


from (x1, y1) to the pair of lines represented
by the equation, ax2 + 2hxy + by2 = 0 is
2
ax 1 2hx 1 y1 by 12
.
(a b ) 2 4 h 2

51

(d) Any second degree curve through the four point


of intersection of f(xy) = 0 & xy = 0 is
given
by f(xy) + xy = 0 where f(xy) = 0 is also a second
degree curve.

(ii) If h = 0, the bisectors are xy = 0


i.e. x = 0, y = 0
(iii) The two bisectors are always at right angles,
since we have
coeff. of x2 + coeff. of y2 = 0
(g) Pair of straight lines perpendicular to the lines
ax2 + 2hxy + by2 = 0 and through origin are
given by bx2 2hxy + ay2 = 0.

(e) A homogeneous equation of degree n represents


n straight lines passing through origin.
(f) The combined equation of angle bisectors
between the lines represented by homogeneous
x2 y2
equation of 2nd degree is given by
=
ab
xy
, a b, h 0.
h
(i) If a = b, the bisectors are x2 y2 = 0
i.e. x y = 0, x + y = 0

SOLVED EXAMPLE
Ex.1

Sol.

Change to Cartesian coordinates the equations


1
1

.
(1) r = a sin , and (2) 2 = 2 cos
a
r
2
(1) Multiplying the equation No. 1 by r, it
becomes r2 = ar sin ,
i.e. by using the relations b/w cartesian and
polar coordinate
the equation becomes, x2 + y2 = ay. Ans.
(2) Squaring the equation (2), it becomes

a
r = a cos
=
(1 + cos), now by using
2
2
the relation between cartesian and polar
coordinate. The equation becomes
i.e. 2r2 = ar + ar cos,
2

i.e. 2(x2 + y2) = a


2

If the segments joining the points A(a, b) and


B(c, d) subtend an angle at the origin, prove that
cos =

Sol.

ac bd
2

(a b 2 ) (c 2 d 2 )

Let O be the origin. Then OA2 = a2 + b2,


OB2 = c2 + d2 and AB2 = (c a)2 + (d b)2
Using cosine formula in OAB, we have
AB2 = OA2 + OB2 2 . OA .OB cos
(c a)2 + (d b)2 = a2 + b2 + c2 + d2
2

a 2 b2

c2 d 2

i.e. (2x + 2y ax) = a (x + y2).

cos

c2 + a2 2ac + d2 + b2 2bd
= a2 + b2 + c2 + d2 2

a 2 b2

c2 d 2

+ ax,

x 2 y2
2

Ex.2

Ans.

51

cos

and BD = (8 6) 2 (3 7) 2 = 4 16 = 2
5

AD = BC and AC = BD. So ADBC is a


parallelogram.
Now,AB = (6 0) 2 (7 1) 2 = 36 64 = 10
and CD =
2(ac + bd) = 2
cos =
Ex.3

Sol.

a 2 b2

c2 d 2

ac bd
2

(a b 2 ) (c 2 d 2 )

cos
Ans.
Ex.5

If the point P(x, y) is equidistant from the points


A(a + b, b a) and B(a b, a + b), then
(A) ax = by
(B) bx = ay
(C) x2 y2 = 2(ax + by)
(D) P can be (a, b).
We have PA = PB, i.e., (PA)2 = (PB)2.
[x (a + b)]2 + [y (b a)2

Sol.

Ex.6

= [(y b) + a]2 [(y b) a]2


4b (x a) = 4a(y b) bx = ay

( 2a 2a ) 2 ( 4a 6a ) 2

BC =

( 3 a) 2 a 2

Show that four points (0, 1), (6, 7), (2, 3) and
(8,3) are the vertices of a rectangle.
Let A(0, 1), B(6, 7), C(2, 3) and D(8, 3) be the
given points. Then

64 16 4

(8 0) 2 (3 1) 2

BC =

(6 2) 2 (7 3) 2

AC =

( 2 0) 2 (3 1) 2

=2a

3 a) 2 a 2

= 2a

If t1, t2 and t3 are distinct, the points (t1, 2at1 + at 13 ),


(A) t1 t2 t3 = 1
(B) t1 + t2 + t3 = t1 t2 t3
(C) t1 + t2 + t3 = 0
(D) t1 + t2 + t3 = 1

Sol.

The given points are collinear if = 0

t1
t2
t3
t1
t2
t3

AD

= 2a

(t2, 2at2 + at 32 ), (t3, 2at3 + at 33 ) are collinear if

....(1)
Ans.[B, D]
Therefore, (B) is correct. Also, P(a, b) satisfies the
condition (1), so that P can be (a, b) and hence (D) is
also correct.

Sol.

AB =

Since AB = BC = CA, the triangle is equilateral


and hence acute-angled.
Ans.[A, B]

= [(x a) + b]2 + [(y b) a]2


[(x a) + b]2 [(x a) b]2

Ex.4

= 10.

Clearly,
AB2 = AD2 + DB2 & CD2 = CB2 + BD2.
Hence, ADBC is a rectangle.
Ans.
The points A(2a, 4a), B(2a, 6a) and
C(2a + 3 a, 5a)(a > 0) are the vertices of
(A) an acute-angled triangle
(B) an equilateral triangle
(C) an isosceles obtuse angled triangle
(D) a right-angled triangle.

and CA =

= [x (a b)]2 + [y (a + b)]2
[(x a) b]2 + [(y b) + a]2

(8 2) 2 (3 3) 2

64 16 = 4

= 2

4 16

2 t 1 t 13
2 t 2 t 32
2 t 3 t 33

2at 1 at 13
2at 2 at 32
3at 3 at 33

1
1= 0 a
1

1
1 =0
1

Applying R2 R2 R1, R3 R3 R1, we get

t1
t 2 t1
t 3 t1

2 t1 t 13
1
3
3
2( t 2 t 1 ) ( t 2 t 1 ) 0 = 0
2( t 3 t 1 )( t 33 t 13 ) 0

51

t1
(t2 t1)(t3 t1) 1
1

2 t 1 t 13
2 t 22 t 12 t 2 t 2
2 t 32 t 12 t 3 t 1

BC2 = (x2 x3)2 + (y2 y3)2 a positive integer.

1
0 =0
0

If the triangle is equilateral,


then AB = BC = CA = a (say) and
A = B = C = 60.

(t2 t1) (t3 t1) (t3 t2) (t3 + t2 + t1) = 0


t1 + t2 + t3 = 0 [ t1 t2 t3]
Ex.7

Sol.

Area of the triangle = (1/2) bc sinA

Ans. [C]

= (1/2) a2 sin 60 = (a2/2). ( 3 / 2) = ( 3 / 4) a2


which is irrational,
a2 is a positive integer.

The points (k, 2 2k), (k + 1, 2k) and


(4 k, 6 2k) are collinear for
(A) all values of k
(B) k = 1
(C) k = 1/2
(D) no value of k.
The given points are collinear if
k
k 1
4k

2 2k
2k
6 2k

2k 1
4 2k

Now, the area of the triangle in terms of the


coordinates
= (1/2) [x1 (y2 y3) + x2 (y3 y1) + x3 (y1 y2)]
which is a rational number.
This contradicts that the area is an irrational
number, if the triangle is equilateral.
Ans.

1
1 =0
1

2 2k

4k 2
4

0 =0
0

Ex.10

of any point P if PA = PB and Area of PAB = 10.

[R2 R2 R1, R3 R3 R1]

Sol.

4(2k + 1) (4 2k) (4k 2) = 0

Let the coordinates of P be (x, y). Then


PA = PB PA2 = PB2

(1 2k) (4 8 4k) = 0
(1 2k) (k + 1) = 0 k = 1

(x 3)2 + (y 4)2 = (x 5)2 + (y + 2)2


Ans.[B]

x 3y 1 = 0

Note : for k = 1/2 first two points are identical.


Ex.8

Sol.
=

45
2
where A(1, 3), B(2, 5), C(3, 1), D(0, 2) and E(2,
t). Here the points are given in order.
The area of the pentagon ABCDE

y
4
2

1
1 = 10
1

6x + 2y 46 = 0 or 6x + 2y 6 = 0
3x + y 23 = 0 or 3x + y 3 = 0 ....(ii)

3
5

1
(5 6) (2 15) (6 0) (0 4) (6 t )
2

1
1
(11 17 6 4 6 t ) =
|44 t|
2
2

Ex.11

44 t = 45 ; t = 44 45 = 1, 89.Ans.

Sol.

x
3
5

6x + 2y 26 = 20

45
=
2

Ex.9

...(i)

Now, Area of PAB =10

Find t if the area of the pentagon ABCDE be

1 1

2 2

If the coordinates of two points A and B are


(3, 4) and (5, 2) respectively. Find the coordinates

If the vertices of a triangle have integral


coordinates, prove that the triangle cannot be
equilateral.
Let A (x1, y1), B (x2, y2), C (x3, x3) be a
triangle and x1, x2, x3, y1, y2, y3 be integers.

Sol.

Solving x 3y 1 = 0 and 3x + y 23 = 0
we get x = 7, y = 2.
Solving x 3y 1 = 0 and 3x + y 3 = 0, we get
x = 1, y = 0.
Thus the coordinates of P are (7, 2) or (1, 0).
Ans.
The line x + y = 1 meets xaxis at A and
y-axis at B. P is the mid-point of AB (Fig.) P 1 is
the foot of the perpendicular from P to OA ; M1 is
that of P1 from OP ; P2 is that of M1 from OA; M2
is that of P2 from OP; P3 is that of M2 from OA and
so on. If Pn denotes the n th foot of the
perpendicular on OA from Mn1, then OPn =
Let x + y = 1 meets x-axis at A (1, 0) and
y-axis at B (0, 1)

51

Also, area of the ABC is given by

The coordinates of P are (1/2, 1/2) and PP1 is


perpendicular to OA.
OP1 = P1P = 1/2
Equation of line OP is y = x.

1
2

1 (y 2

y3 )

1
2

1 3k 5
5k 1

5k 1

(7) 2
5
7

2 k 1
k

1
k

x 1 ( y 2 y 3 ) x 2 ( y 3 y1 ) x 3 ( y1 y 2 )
2.

1

2
5k 1
3k 5

5k 1

(7 ) 2
5 2
7

1
k

1
k

We have
OM 2n 1 = OPn2

+ Pn M 2n 1 = 2 OP 2n = 2p 2

(say)

OP 2n

Also,

OM 2n 1

Pn 1

1 =

1 2
p n 1
2
1 2
1
2
p n p n 1 p n p n 1
4
2
1
1
OPn = Pn =
pn1 = 2 p n 2 = ........
2
2
1
1
.. = n 1 p1 n
Ans.
2
2
2
= 2p n

Ex.12
Sol.

Ex.14

p n 1 M 2n 1

Show that the area of the triangle with vertices


[(a + 1) (a + 2), [a + 2)], [(a + 2) (a + 3), [a + 3)] and
[(a + 3) (a + 4), (a + 4)] is independent of a.
The area of the given triangle is
(a 1)(a 2)

a2

(a 3)(a 4)

a 3
a4

1
1

1
(a 2)(a 3)
=
2

Sol.

14k66 = 4k + 4, 10k = 70, k = 7


or 14k 66 = 4k 4, 18k = 62,
k = (31/9).
Therefore value of the k = 7, 31/9
Ans.
If the coordinates of the mid-points of the sides of
a triangle are (1, 2) (0, 1) and (2, 1). Find the
coordinates of its vertices.
Let A(x1, y1), B(x2, y2) and C(x3, y3) be the vertices
of ABC. Let D(1, 2), E(0, 1), and F(2, 1) be
the midpoints of sides BC, CA and AB
respectively.
Since D is the midpoint of BC.

x2 x3
y y3
= 1, and 2
=2
2
2

x2 + x3 = 2 and y2 + y3 = 4

....(i)

Applying R2 R2 R1 and R3 R3 R2, we get

1
=
2

(a 1)(a 2)

a2

2( a 2)
2(a 3)

1
1

Which is independent of a.
Ex.13

Sol.

1
0 = | 1| = 1
0

Ans.

Similarly, E and F are the mid-points of


CA and AB respectively.

x1 x 3
y y3
= 0 and 1
= 1
2
2

The point A divides the join of P (5, 1) and Q =


(3, 5) in the ratio k : 1. Find the two values of k for
which the area of
ABC where

x1+ x3 = 0 and y1 + y3 = 2

B (1, 5), C (7, 2) is equal to 2 square units.


Co-ordinates of A, dividing the join of P (5, 1)

x1+ x2 = 4 and y1 + y2 = 2
...(iii)
From (i), (ii) and (iii), we get
(x2 + x3) + (x1 + x3) + (x1 + x2) = 2+ 0 + 4
and (y2 + y3) + (y1 + y3) + (y1 + y2) = 4 2 2

and Q (3, 5) in the ratio k : 1 are given by


(3k 5/k +1, 5k + 1/k +1)

and

...(ii)

x1 x 2
y y2
= 2 and 1
= 1
2
2

51

x1+ x2 + x3 = 3 and y1 + y2 + y3 = 0

...(iv)

From (i) and (iv), we get x1 + 2 = 3


and y1 + 4 = 0
x1 = 1 and y1 = 4
So, the coordinates of A are (1, 4)
From (ii) and (iv),
we get x2 + 0 = 3 and y2 2 =0

so that 1 3 =

1 3 = 22
2 =

So, coordinates of B are (3, 2)


From (iii) and (iv),
we get x3 + 4 = 3 and y3 2 = 0

1
1
ab 2 = 22 2 =
ab
2
2

1
1
ab (sin2 cos2) =
ab
2
2

sin2 cos2 = 1
That is, = (2m + 1)/2, m I. But for this value

x3 = 1 and y3 = 2
So, coordinates of C are (1, 2)
Hence, the vertices of the triangle ABC are
A(1, 4), B(3, 2) and C(1, 2).
Ans.
If 1 is the area of the triangle with vertices
(0, 0), (a tan , b cot ), (a sin , b cos ),

of , the vertices of the given triangles are not


defined.
Hence 1, 2 and 3 cannot be in G.P. for any
value of .
Ex.16

2 is the area of the triangle with vertices


(a, b), (a sec2 , b cosec2 ),
(a + a sin2 , b + b cos2 ),
and 3 is the area of the triangle with vertices
(0, 0), (a tan , b cot ), (a sin , b cos ),
Show that there is no value of for which 1, 2

Sol.

Ans.

The line x + y = a, meets the axis of x and y at A


and B respectively. A triangle AMN is inscribed in
the triangle OAB, O being the origin, with right
angle at N. M and N lie respectively on OB and
AB. If the area of the triangle AMN is 3/8 of the
area of the triangle OAB, then AN/BN =
AN
Let
= .
BN

a
a
,

1 1
where (a,0) and (0,a) are the coordinates of A and
B respectively.
Now equation of MN perpendicular to AB

and 3 are in G.P.


Sol.

1
ab 2 .
2

Now 1, 2 and 3 are in G.P. if

x2 = 3 and y2 = 2

Ex.15

1
ab |sin + cos |
2

Then the coordinates of N are

We have
0

0
b cot
b cos

1
a tan
1 =
2 a sin

1
1
1 =
ab |sin
2
1

cos |

1
and 2 =
2

a
a sec 2
a a sin 2

b
b cos ec 2
b b cos 2

1
1
1

applying C1 C1 aC3 and C2 C2 bC3


we get
0

1
2
2 =
ab tan
2
2
sin

0
cot 2
cos 2

1
1
1

is y

a
So the coordinates of M are 0,
1

Therefore, area of the triangle AMN is

1
ab(sin2 cos2)
2
0

1
a tan
and 3 =
2 a sin

a
a
1
a.
=x
or x y =
1
1
1

0
b cot

1
1

b cos

1
a 2
1 a
2
a

(1 ) 2
(1 ) 2
2 1

Also area of the triangle OAB = a2/2.


So that according to the given condition.

51

a 2
(1 )

OA = OB = OC.

3 1 2
. a
8 2

x12 + x12 tan21 = x 22 + x 22 tan22

32 10+ 3 = 0
= 3 or = 1/3.

= x 32 + x 32 tan2 3
or x 12 sec2 1 = x 22 sec2 2 = x 32 sec2 3

for = 1/3, M lies outside the segment OB and


hence the required value of is 3.
Ans.
Ex.17

A line L intersects the three sides BC, CA and AB


of a triangle ABC at P, Q and R, respectively.
Show that
BP CQ AR
.
.
1
PC QA RB

Sol.

Let A (x1, y1), B(x2, y2) and C (x3, y3) be the


vertices of ABC, and let x + my + n = 0 be
equation of the line L. If P divides BC in the ratio
: 1, then the coordinates of P are

or

x3
x1
x2

= k (suppose)
cos 1 cos 2 cos 3

the vertices of the triangle become

x 3 x 2 y 3 y 2
,

1
1

A = (kcos 1, ksin 1),


B = (kcos 2, ksin 2),

Also, as P lies on L, we have

C = (kcos 3, ksin 3)

x 3 x 2
y 3 y 2

m
n 0
1
1

The centroid G

cos 1 cos 2 cos 3 sin 1 sin 2 sin 3


,k

3
3

We know that the orthocentre H, centroid G and


circumcentre O are collinear.
So, 'm' of HO = 'm' of GO

x 2 my 2 n
BP

x 3 my 3 n
PC

'm' of HO =
....(i)

Similarly, we obtain

x my 3 n
CQ
3
QA
x 1 my1 n
and

x my1 n
AR
1
RB
x 2 my 2 n

....(ii)

B(x2, x2tan2) and C(x3, x3tan3). If the


circumcentre coincides with the origin and the
orthocentre H = ( x , y) prove that
=
Sol.

x (sin1 + sin2 + sin 3 ).

sin 1 sin 2 sin 3


0
3
=
cos 1 cos 2 cos 3
k.
0
3

....(iii)

Vertices of a triangle are A(x 1, x1tan1),

y (cos 1 + cos2 + cos3)

and 'm' of GO

k.

Multiplying (i), (ii) and (iii), we get the required


result.
Ans.
Ex.18

y0 y

x0 x

sin 1 sin 2 sin 3


cos 1 cos 2 cos 3
y sin 1 sin 2 sin 3

x cos 1 cos 2 cos 3

or y (cos 1 cos 2 cos 3 )


=

x (sin1 + sin 2+ sin3). Ans.

Ex. 19 The circumcentre of a triangle having vertices A


(a, a tan ), B (b, b tan ), C (c, c tan ) is at the

Here, the circumcentre O = (0, 0). So

51

origin, where ++ = . Show that the

Sol.

orthocentre lies on the line

Let P be the middle point of the line segment


joining A(3, 1) and B(1, 1).

3 1 1 1
,
= (2, 0).
2
2

Then P =

4 cos

Sol.

cos cos x 4 sin sin sin .y y.


2
2
2
2
2
2

Since circumcentre O' is the origin, and therefore


equation of the circumcircle may be
x 2 + y 2 = r2
....(i)

Let P be shifted to Q where PQ = 2 and


y-coordinate of Q is greater than that of P (from
question).

Since vertex A (a, a tan ) is upon (i), therefore


a2 (1 + tan2 ) = r2 a = r cos
A (r cos , r sin )
Similarly B (r cos, r sin ),

Now,

C (r cos, r sin)

'm' of AB =

1 (1)
2

1.
1 3
2

1
= 1.
1
Coordinates of Q by distance formula
= (2 2cos, 0 2sin) where tan = 1
'm' of PQ =

1
1
= (2 2 , 2 ).
, 0 2.
= 2 2 .
2
2

As y-coordinate of Q is greater than that of P,


Q = (2 + 2 , 2 ). This is the required point.
Ans.

Centroid G

)
r (cos cos cos ) r (sin sin sin Ex.21
ABC is a variable triangle with the fixed vertex
,

.
3
3

C(1, 2) and A,B having the coordinates


Circumcentre O' (0, 0) and orthocentre H (h, k),
(cos t, sin t), (sin t, cos t) respectively where t is
let, Since we know that O',G, H are collinear,
a parameter. Find the locus of the centroid of the
therefore slope of O'G = slope of O' H
ABC.
sin sin sin
k
Sol.
Let G() be the centroid in any position. Then

i.e.
cos cos cos

1 cos t sin t 2 sin t cos t


,

3
3

G() =

Point (h, k) will be upon


x(sin+ sin+ sin) y(cos + cos+ cos) = 0

sin
sin 0
1 4 sin
2
2
2

And hence the result because ++ = ,


Ans.
Ex.20

2 sint cost
1 cos t sin t
,=
3
3
or 3 1 = cos t + sin t,
....(i)
=

x 4 cos cos cos


2
2
2

The middle point of the line segment joining


(3, 1) and (1, 1) is shifted by two units (in the
sense of increasing y) perpendicular to the line
segment. Find the coordinates of the point in the
new position.

3 2 = sin t cos t
....(ii)
Squaring and adding, (i) and (ii) equation
(3 1)2 + (3 2)2 = (cos t + sin t)2 +
(sin t cos t)2
2 (cos2 t + sin2 t) = 2
the equation of the locus of the centroid is
(3x 1)2 + (3y 2)2 = 2
or 9 (x2 + y2) 6x 12y + 3 = 0
3 (x2 + y2) 2x 4y + 1 = 0.
Ans.

51

Ex.22

Sol.

the transformed equation of the pair of lines for


the new axes will be
(a cos2 + h sin 2 + b sin2 )x2

Two points P (a, 0) and Q (a, 0) are given. R is a


variable point on one side of the line PQ such that
RPQ RQP is a 2. Find the locus of R.
Let R (h, k) be the variable point (see Fig.). Then
RPQ = and RQP = , so that = 2. Let

+ {(b a) sin 2 +2h cos 2}xy


+ (a sin2 h sin 2 + b cos2 )y2 = 0.
This equation will not contain xy if
(b a) sin 2 + 2hcos 2 = 0

RM PQ, so that RM = k, MP = a h and MQ =


a + h.
RM
RM
k

Then tan=
= tan =
=
MQ
MP a h

2h
ab
1
2h
=
tan1
.
2
ab
This is the required angle.
or tan 2=

k
ah

Ex.24
Sol.

Therefore, from 2 = , we have

Sol.

What can be said regarding a line if its slope is (i)


positive, (ii) zero, (iii) negative Let be the angle of inclination of the given line
with the positive direction of x-axis in
anticlockwise sense. Then, its slope is given by
m = tan .

tan 2 = tan ( )

(i) Slope is positive m = tan > 0 lies

tan tan
k ( a h ) k (a h )

=
1 tan tan
a2 h2 k2

(ii) Slope is zero m = tan = 0 = 0.

a2 h2 + k2 2hk cot 2
Hence, the locus of R (h,k) is
x2 y2 + 2xy cot 2 a2 = 0.
Ex.23

Ans.

between 0 and 90. is an acute angle.


either the lines is x - axis or it is parallel to
x-axis.
(iii) Slope is negative m = tan < 0
lies between 90 and 180
is an obtuse angle.
Ans.

Ans.

The equation of a pair of straight lines is


ax2 + 2hxy + by2 = 0. By what angle must the axes
be rotated so that the term containing xy in the
equation may be removed ?
Here the origin remains fixed. Let the axes be
turned about the fixed origin through an angle in
the anticlockwise sense and new coordinates of the
point (x, y) become (x', y').
Then the equations of transformation will be
x = x' cos y' sin
y = x' sin + y' cos

Ex.25

Sol.

The sum of the reciprocals of the intercepts made


on the coordinate axes by any line not passing
through the origin and through the point of
intersection of the lines 2x + 3y = 6 and 3x + 2y = 6
is constant.
Equation of any line through the points of
intersection of the given lines is
2x + 3y 6 + k (3x + 2y 6) = 0
(2 + 3k) x + (3 + 2k) y 6 (k + 1) = 0

y
x

6(k 1) = 1, when k 1
6(k 1)
2 3k
3 2k

the changed equation will be


a(x' cos y' sin )2 + 2h
(x' cos y' sin )(x' sin + y' cos )
+ b (x' sin + y' cos ) = 0
2

or (a cos + 2h sin cos + b sin2 ) x'2


2

+ (a sin 2 + 2h cos 2 + b sin 2) x' y'


+ (a sin2 2h sin cos + b cos2 ) y'2 = 0

and in this case sum of the reciprocals of the


intercepts made by this line on the coordinate axis
is equal to
2 3k 3 2k 5(k 1) 5

.
6(k 1)
6(k 1) 6
However, for k = 1, the line becomes x = y which
passes through the origin.
Ans.

51

Ex.26

Sol.

In what ratio is the line joining the point (2, 3) and


(4, 1) divides the segment joining the points (1, 2)
and (4, 3)
The equation of the line joining the points (2, 3) and
(4, 1) is
1 3
y3=
(x 2)
42
y3=x+2x+y5=0
...(i)
Suppose the line joining (2, 3) and (4, 1) divides
the segment joining (1, 2) and (4, 3) at point P in
the ratio : 1. Then the coordinates of P are

Line to L passing through (1/2, 0) is


x y = or x y = 1/2
...(ii)
y-axis x = 0
...(iii)
Solving them in pairs, we get the points
1

5 3
,
(0, 2), 0, and
2

4 4
Its area =
Ex.29

4 1 3 2
,

1 1
Clearly P lies on (i),
4 1 3 2

5 0 1.

1
1
Hence the required ratio is : 1 i.e. 1 : 1 Ans.
Ex.27

Sol.

or

2p

50
3

Sol.

Find the area of the triangle formed by y-axis, the


straight line L passing through the points
(1, 1) and (2, 0) and the straight line perpendicular to
the line L and passing through the points (1/2, 0)
and show that it is 25/16 sq. units.
The line L is x + y = 2
...(i)

25
16

Ans.

sin 2 2
p
p

, 0 , B 0,

A
sin
cos

Lines through A, B parallel to axes are


p
p
,y
cos
sin

x=

These meet the line through origin and to given


line i.e., x sin y cos = p in P and Q.

p
p sin
p cos p
,
,
,Q

2
cos cos
sin 2 sin

PQ2 (by distance formula)

= p2 cos2 2

sin cos

= 16p2 cos2 2
PQ =
Ex.30

or p2 = 25 p = 5.

3
Hence from (i), the equations of required lines are
x 3 + y = 10.
Ans.

Ex.28

Sol.

The line x cos + y sin = p meets the axes of coordinates at A and B respectively. Through A and
B lines are drawn parallel to axes so as to meet the
perpendicular drawn from origin to given line in P
and Q respectively; then show that
| PQ | =

4p | cos 2 |

Find the equation of the straight line on which the


perpendicular from origin makes an angle of 30
with x-axis and which forms a triangle of area 50
3 with the coordinate axes.
Let p be the length of the perpendicular drawn
from the origin on the required straight line. Given
that the line make an angle = 30 with x-axis.
The equation of the line is
x cos 30 + y sin 30 = p
or x ( 3 / 2) + y/2 = p
...(i)
If the line (i) meets the coordinates axes at A and B
respectively, then putting y = 0 in (i),
x = 2p/ 3 = OA
and putting x = 0 in (i), y = 2p = OB.
Now area of right angled OAB
50
1
=
OA. OB =
(Given)
2
3

1
1 5
2
.
2
2 4

Sol.

1
4

cos sin

1
(2 sin cos ) 4

4p | cos 2 |

Ans.

sin 2 2

The equations of two sides of a square are


3x + 4y 5 = 0 and 3x + 4y 15 = 0. The line
along the third side has a point (6, 5) on it. Find
the equations of this and the remaining side of the
square.
Clearly 3x + 4y 5 = 0 and 3x + 4y 15 = 0 are
two parallel lines, their 'm' being equal to
3/4. Let these sides be AB and DC in the square
ABCD and the point P (6, 5) be on the side AD.

51

Solving (iii) and (i), co-ordinates of A (0, 1)

The side AD is on a line through P (6, 5)


perpendicular to 3x + 4y 15 = 0 any line
perpendicular to 3x + 4y 15 = 0 has the equation
4x 3y = .
It passes through (6, 5). So = 4 6 3 5

7 1
,
8 8

Solving (i) and (ii) B

7 13
,
6 6

Solving (ii) and (iii) C

= 9
the equation of AD is 4x 3y = 9

To obtain the coordinates of incentre use

ax1 bx 2 cx 3 ay1 by2 cy3


,

abc
abc

As BC || AD, the equation of BC is 4x 3y =


Let PQ BC
Then PQ =

4.6 3.5
4 2 (3) 2

9
5

= distance between the parallel lines AB and DC.


5

Now, any point on AB is 0,


4

The distance of this point from DC


=

5
15
4
32 4 2

3 0 4

Hence,

9
5

2 and

2 ; or

, 1 .
12

we get incentre as
Ex.32

Sol.

Ans.

A vertex of an equilateral triangle is (2, 3) and the


equation of the opposite side is x + y = 2. Find the
equations of the other two sides and the length of
each side of the triangle.
Other sides pass through the vertex (2, 3).
the equations of other sides will be of the form

| 9 | = 10

9 = 10 = 9 10 = 1, 19
The equation of the remaining side BC is
4x 3y = 1 or 4x 3y = 19.
Note : As it has not been verified whether P lies
between A and D or not, use of PQ = AD = PA +
PD may lead to wrong result.
Ans.
Ex.31
Sol.

Find the incentre of the triangle whose sides are


x + y = 1, x y + 1 = 0, 7x y = 6.
Given lines of ABC as
AB : x + y 1 = 0
...(i)
BC : 7x y 6 = 0
...(ii)
CA : x y + 1 = 0
...(iii)

y 3 = m (x 2)
...(i)
The equation of the third side is
x+y=2
...(ii)
1
'm' of (ii) is
, i.e., 1
1
Being equilateral triangle, the angle between (i)
and (ii) will be 60.
tan 60 =
or

3 =

m ( 1)
1 m ( 1)
m 1
1 m

(by angle formula)

51

m 1
=
1 m

m+1=
or (1 +
=

3 (1 m),

3 )m=

3 1 and (1

3 1
3 1

3 )m

3 1

( 3 1) ( 3 1)
,
2
2
3 ,2+

If is angle ABC, then


4 3 / 4
19

tan =
1 ( 4)(3 / 4)
8

3 1

=2

3 (1 m)

or m =
=

Given that AB = AC
ACB = ABC =
If slope of AC is m, then
m3/ 4
19
tan =
=
1 m.(3 / 4)
8

the equations of the other two sides are


y 3 = (2

3 ) (x 2) and

y 3 = (2 + 3 ) (x 2)
or y = (2
y = (2 +

3 )x1+2

3 )x12

3 and

or 19 (4 + 3m) = 8 (4m 3)
m = 4 or 52/89
But 4 is the slope of line AB,
Slope of AC = m = 52/89
Hence, the equation of line AC which passes
through A (2, 7) and have slope m = 52/89 is
y + 7 = (52/89) (x 2)
or 52x + 89y + 519 = 0.
Ans.

3 .

Solving one of these, say,


y = (2 3 ) x 1 + 2 3 with x + y = 2
we get
3 2 3

3 3

x =

32

3 1

y = 2 x = 2

32
3 1

32
,
3 1

B or C =

3
3 1

Ex.34

3
3 1

each side = AB or AC

=
=
=
Ex.33

Sol.

3
3 1

3 2
3 1

2 3 3

3 1

3
3 1

Sol.

1
( 3 ) 2 (2 3 3) 2
3 1
1
3 1

6( 3 1) 2

Find equations of the sides of the triangle having


(3, 1) as a vertex, x 4y + 10 = 0 and
6x + 10y 59 = 0 being the equations of an angle
bisector and a median respectively drawn from
different vertices.
Let the vertices of the triangle be A(3, 1), B(x 1,
y1) and C(x2, y2).Equation of the median through B
be 6x + 10y 59 = 0 and the equation of the angle
bisector
from
C
be
x 4y + 10 = 0
x2 4y2 + 10 = 0
...(i)

x 2 3 y 2 1
,
, the mid-point of AC
2
2

Also D
Ans.

A line 4x + y = 1 through the point A (2, 7) meets


the
line
BC,
whose
equation
is
3x 4y + 1= 0 at the point B. Find the equation to
the line AC, so that AB = AC.
Equation of line AB is 4x + y = 1.
Its slope = 4.
Slope of line BC, 3x 4y + 1= 0 is 3/4

lies on the median through B,


i.e., 6x + 10y 59 = 0

x2 3
y 1
10 2
59 = 0
2

3x2 + 5y2 55 = 0
...(ii)
Solving (i) and (ii) we get x2 = 10, y2 = 5, i.e., the
coordinates of C are (10, 5) and thus the equation
of AC is 6x 7y = 25

51

Let r, r1, r2, ... rn be the distances of the points, R,


R1, R2, ... Rn from O OR = r and ORi = ri
Then coordinates of R are (r cos , r sin ) and of
Ri are (ri cos, ri sin), i = 1, 2, ..., n.
Since Ri lies on aix + biy + ci = 0
ai ri cos + bi ri sin + ci = 0
for i = 1, 2, ..., n.
Let the slope of BC be m, then since. BC and
AC are equally inclined to the angle bisector
x 4y + 10 = 0. We have

6 1
1

m
7
4 1 4m 17
4

1
6 1
4m
34
1 m 1
4
7 4

2
9
so that equation of BC is
2
y5=
(x 10)
9
or 2x + 9y = 65
Since B lies on BC and the median
6x + 10y 59 = 0 through B, we have
2x1 + 9y1 = 65
6x1 + 10y1 = 59
Solving these equations we get
x1 = 7/2, y1 = 8
Hence equation of AB is
8 1
y+1=
(x 3)
7 / 23
18x + 13y = 41

Sol.

i 1

1

ri

n

r

m=

Ex.35

ai
b
1
cos i sin , i = 1, 2, ..., n.
ci
ci
ri
n

i 1

i 1

ai

c i

cos

ai
cos
c i

i 1

bi

i 1

bi
sin
c i

sin

(Given)

ai
ci

i 1

bi

i 1

r cos

r sin n 0

Hence the locus of R is

i 1

ai
ci

bi

i 1

yn0

which is a straight line.


Ex.36

Sol.

For what values of the following three lines are


concurrent
x + y = 1, x + 2y = 3, 2 x + 4y + 9 = 0
The three lines will be concurrent if
1

Ans.

Given n straight lines and a fixed point O, a


straight line is drawn through O meeting lines in
the points R1, R2, R3, ...., Rn and on it a point R is
taken such that
n
1
1
1

...
OR OR1 OR 2
OR n
Show that the locus of R is a straight line.
Let equations of the given lines be aix + biy + ci = 0, i
= 1, 2, ... n. and the point O be the origin (0, 0).
Then equation of the line through O can be written
as
y
x

r,
cos sin
Where is the angle made by the line with the
positive direction of x-axis and r is the distance of
any point on the line from the origin O.

Ans.

or

1
2
4

1
3
9

=0

0
3

0
1

1
3

2 9

13

= 0,

C1 C1 + C3 , C2 C2 + C3
or 13( 3) + 2 + 9 = 0
or 2 + 13 30 = 0
2
= 13 13 4.30 13 289
2
2
13 17
=
= 2, 15
Ans.
2

Ex.37

A variable straight line drawn through the point of


inter-section of lines

x y
x y
1 &
1,
a b
b a

51

Sol.

or y(cos sin ) x(cos + sin ) = 0 ...(i)

meets the co-ordinate axes in A and B. Show that


the locus of the mid-point of AB is the curve 2xy
(a + b) = ab (x + y).
The equation of the variable line through the point
of inter-section of the given lines is of the type :
x y

y y

1 + k
1 = 0

a b

b a

1 k
1 k
x + y = (1 + k)
a b
b a
or (ak + b) x + (bk + a) y = ab (1 + k)
Points A and B are
or

The other diagonal AC will be perpendicular to


(i) and pass through the point A hence its equation

ab(1 k )
ab(1 k )

, 0 and 0,

.
bk a
ak b

Let P (x1, y1) be the mid-point of AB.


ab(1 k )
ab(1 k )
Then x1 =
and y1 =
...(i)
2(ak b)
2( bk a )

is y(cos + sin ) + x(cos sin ) = k


where k = a.
Ex.39

(Observe that the required equation of the locus of


Sol.

1 1 2( a b )
the same as
),
x y
ab

therefore we write (i) in the form.

2(ak b)
1
1
2( bk a )

,
x 1 ab(1 k )
y1
ab(1 k )

Ex.38

where tan = m
As A is on the line x + y 5 = 0,
1 + r1 cos + 2 + r1 sin 5 = 0
or r1 (cos + sin) = 2

1
cos sin
=
r1
2

...(ii)

Ans.

One side of a square makes an angle with


x-axis and one vertex of the square is at the origin.
Prove that the equations of its diagonals are y (cos
sin ) = x (sin + cos ) and

Sol.

Find the equation of the line passing through


the point P(1, 2) cutting the lines x + y 5 = 0 and
2x y = 7 at A and B respectively such that
the harmonic mean of PA and PB is 10.
Let the equation of the line passing through
P(1, 2) be y 2 = m (x 1)
...(i)
Let PA = r1, PB = r2. Then
A = (1 + r1cos, 2 + r1sin)
B = (1 + r2 cos, 2 + r2sin),

1
1 2(a b)(1 k ) x1 y1 2(a b)

x1 y1
ab(1 k )
x1y1
ab
(This eliminates parameter k).
Hence the locus of P (x1, y1) is
2xy (a + b) = ab (x + y).

Ans.

y (sin + cos ) + x (cos sin ) = a, where a is


length of the side of the square.
Let the side OA make an angle with x-axis and
since the side of the square OABC is a, therefore
coordinates of point A are (a cos, a sin). Now
the diagonal OB will make an angle of 45 +
with x-axis and pass through origin O. Hence its
equation is
y = tan (45 + ) x
or y =

1 tan
x
1 tan

As B is on the line 2x y 7 = 0,
2 (1 + r2cos) (2 + r2 sin) 7 = 0
or r2(2cos sin) = 7

1 2 cos sin

r2
7

...(iii)

Now, HM of PA, PB is 10
10 =
or

2PA . PB
;
PA PB

r1 r2 1
;
r1 r2
5

10 =

2r1 r2
r1 r2

1 1 1

r2 r1 5

51

2 cos sin (cos sin )


1
+
=
5
7
2
[from (ii), (iii)]
or 10 (2cos sin) + 35 (cos + sin) = 14
or

or 55cos + 25 sin = 14
or

55 2 25 2 cos( ) = 14

cos =

Where

55
55 2 25 2

or cos ( ) =
= + cos1
= cos1

14
5 146
14
5 146

11

+ cos1

5 146

14
5 146

from (i), the equation of the line is

11
14
1
(x
cos1
y 2 = tan cos
146
5 146

1)
Ans.

Ex.40

Sol.

The vertices of a triangle OBC are O (0, 0),


B (3, 1), C (1, 3). Find the equation of the
line parallel to BC and intersecting the sides OB
and OC and whose perpendicular distance from
the point (0, 0) is 1/2.
Here B (3, 1) and C is (1, 3).
3 ( 1)
Slope of BC =
= 1.
1 ( 3)
Hence any line parallel to BC will have its
slope = 1
Hence its equation is
y = x + c or x + y c = 0
Its distance from origin is 1/2

the axes will be ive. Therefore we should choose


+ sign out of .
Hence the required line is
x + y + 2 /2 = 0 or 2x + 2y + 2 = 0 .
Ans.
Ex.41 A triangle has the line y = m 1x and y = m2x for two
of its sides, with m1 and m2 being roots of the
equation bx2 + 2hx + a = 0. If H (a, b) is the orthocentre of the triangle, show that the equation of the
third side is
(a + b) (ax + by) = ab(a + b 2h).
Sol.
The given lines y = m1x and y = m2x intersect at
the origin O(0, 0). Thus one vertex of the triangle
is at the origin O. Therefore let OAB be the
triangle and OA, OB be the lines
y = m1x
...(i)
and y = m2x
...(ii)
respectively.

Let the equation of the third side AB be


y = mx + c
...(iii)
Given that H (a, b) is the ortho-centre of the
OAB,
OH is to AB
(b/a) m = 1 m = a/b
...(iv)
Solving (iii) with (i) and (ii), the coordinates of A

cm

1
,
are
and those of B are
m

m
m

1 m
1

c
cm2

,
m2 m m2 m

(c)/( 1 1 ) =
c =

1
2

2/2

Required equation is x + y 2 /2 = 0
Now the lines OB and OC are in 3rd quadrant.
This line meets both OB and OC and hence it will
also be in 3rd quadrant, so will the intercepts on

Now equation of line through A and perpendicular


to OB is
y

cm1
1

m1 m
m2

or y =

m1 m

x
c(m1m 2 1)

m 2 m 2 (m1 m)

...(v)

51

Similarly, equation of line through B and


perpendicular to OA is
y=

x c(m1m 2 1)

m1 m1 ( m 2 m)

...(vi)

The point of intersection of (v) and (vi) is the


ortho-centre H (a, b).
Subtracting (vi) from (v), we get
x=a=

cm(m1m 2 1)
(m1 m)(m 2 m)

[m 1 m 2 m(m 1 m 2 ) m 2 ]a
or c =
...(vii)
m(m 1 m 2 1)
Since m1 and m2 are the roots of the equation
bx2 + 2hx + a = 0
m1 + m2 = 2h/b and m1m2 = a/b
From (vii), we have
c=
=

[a / b 2hm / b m 2 ]a
m(a / b 1)

[a 2hm bm 2 ]a
m (a b)

where vertices are (x1, y1), etc.


0 4 4 33 0
,
= (0, 0)
G=
3
3

To find the circumcentre


Let M () be the circumcentre. Then
MA = MB = MC
( 0)2 + ( + 3)2 = ( 4)2 + ( 3)2
= ( + 4)2 + ( 0)2
or 2 + 2 + 6 + 9 = 2 + 2 8 6 + 25
= 2 + 2 8 + 16

6 + 9 = 8 + 16
or 8 + 12 16 = 0,
2

y = mx

(a 2hm bm )a
m1 (a b)

or y =

a
(a 2ha / b b.a 2 / b 2 )a
x
b
(a / b)(a b)

or (ax + by) (a + b) = ab (a + b 2h).

Sol.

x1 x 2 x 3 y1 y 2 y3
,

3
3

or 6 + 9 = 8 6 + 25 = 8 + 16
6 + 9 = 8 6 + 25 and

From (iii), the equation of third side AB is

Ex.42

Solving (i), (iii) we get y = 3, x = 0;


A = (0, 3)
To find the centroid
We know that centroid G

i.e., 2 + 3 4 = 0
and 8 6 + 7 = 0

Ans.

Find the coordinates of the centroid, circumcentre


and orthocentre of the triangle formed by the lines
3x 2y = 6, 3x + 4y + 12 = 0 and
3x 8y + 12 = 0.
Let the sides AB, BC and CA have the equations
3x 2y 6 = 0
...(i)
3x 8y + 12 = 0
...(ii)
3x + 4y + 12 = 0
...(iii)

Solving (i) and (ii) we get =

1
23
,
12
18

1 23
,

12 18

circumcentre =

To find the orthocentre H


MG
1
Use =
=
GH
2
Let H () be the orthocentre.
0=

2.

1
1
12 =

0=

2.

... (i)

3
respectively.
Solving (ii), (iii) we get y = 0, x = 4;
C = ( 4, 0)
Solving (i), (ii) we get y = 3, x = 4;
B = (4, 3)

...(i)
...(ii)

23
23
18 =

... (ii)

1
23

H : ,
6
9

Ans.

51

Ex.43 A ray of light is sent along the line


x 2y + 5 = 0; upon reaching the line
3x 2y + 7 = 0, the ray is reflected from it. Find
the equation of the line containing the reflected
ray.
Sol.
A ray of light is sent along the line (say AP) whose
equation is
x 2y + 5 = 0,
...(i)
Equation of another line is
3x 2y + 7 = 0.
...(ii)
Solving (i) and (ii), we get P(1, 2) as their point
of intersection.
Thus if AP is the incident ray and (ii) is the surface
then the reflected ray will be along the line PB,
such that
APN = BPN = (say),

Sol.

other two vertices also move on two perpendicular


lines.
The slope of the given lines is b/a and the two
vertices are clearly on the diagonal BD of the
square ABCD. If m be the slope of the line
inclined at an angle of 45 to BD, then

b
a =1
tan ( 45) =
b
1 m
a
m

Where PN is normal to the surface (st. line) given


by (ii)

(a b )
a b
or
(a b )
ab

m =

AB is y 0 =

ab
(x a)
ab

ab
(x 0)
a b
Solving these by subtracting etc. the point A is
AD is y b =

Now slope of AP = 1/2, slope of (ii) = 3/2


slope of PN normal to (ii) = 2/3

ab ba
,

, C is obtained by using the fact


2
2

Let m be the slope of AB.


tan APN = tanBPN

that mid-point of AC and BD is same.

m ( 2 / 3)
1 / 2 (2 / 3)
or
=
1 (1 / 2)(2 / 3)
1 m( 2 / 3)
or 7(3 2m) = 4 (3m + 2)
m = 1/2 or 29/2
But 1/2 is the slope of line AP,
slope of reflected ray (line PB) = 29/2

ab ab
,

2
2

C is

2nd part
The opposite vertices B, D move on two
perpendicular lines x-axis and y-axis. Now the

Equation of line PB (reflected ray) which


passes through P (1, 2) is
y 2 = (29/2)(x + 1)
or 29x 2y + 33= 0.
Ans.
Ex.44

ab ba
,
lies on y = x and point
2
2

point A

ab ab
,

2
2

One diagonal of a square is the portion of the line

x y
1 which is intercepted between the axes.
a b
Find the co-ordinates of other two vertices of the
square. Also prove that if two opposite vertices of
a square move on two perpendicular lines, the

Ex.45

lies on y = x. Both these lines

are clearly perpendicular.


The three sides of a triangle are

Ans.

Lr xcos r + ysin r pr = 0; r = 1, 2, 3. Show


that the orthocentre of the triangle is given by
L1cos(2 3) = L2 cos(3 1) = L3 cos(1 2).

51

Sol.

Let the equations of BC, CA, AB be respectively


L1 = 0, L2 = 0, L3 = 0
Any line passing through A, i.e.,
the intersection of the lines L2 = 0, L3 = 0 is

(ii) If a, b, c are in A.P., then the straight lines


ax + by + c = 0 will always pass through the
point.

Sol.

The given equation can be put in the form


(i) a (x + y 1) + b (2x + 3y 1) = 0
or (x + y 1) + (2x + 3y 1) = 0
where = b/a
Above is of the form P + Q = 0 and as such it

L2 + L3 = 0

represent a line through the intersection of the line


P = 0 and Q = 0
i.e. x + y 1 = 0 and 2x + 3y 1 = 0.
Solving the above, we get the point (2, 1) which
is fixed.
(ii) 2b = a + c or c = 2b a

...(i)

or x(cos2 + cos 3) + y(sin 2 + sin 3)


(p3 + p3) = 0
'm' of this line =

cos 3 cos 3
sin 2 sin 3

ax + by + (2b a) = 0

cos 1
It is to BC whose 'm' =
sin 1

or a (x 1) + b (y + 2) = 0
or (x 1) + (y + 2) = 0

cos 2 cos 3 cos 1


= 1
.
sin 2 sin 3 sin 1

or cos1 (cos2 + cos3) + sin1 (sin2 + sin3) = 0

It passes through the intersection of x 1 = 0 and


y + 2 = 0 i.e. the point (1, 2).
Ex.47

or cos(1 2) + cos (3 1) = 0
=

cos(1 2 )
cos( 3 1 )

satisfying the condition that p12 = p 22 + 3. Prove


that the product of the distances of the same point
from the lines x + y + 2 = 0 & x y 3 = 0 varies as
its distance from the line x 3y 1 = 0.

Putting in (i), the equation of the altitude through


A becomes
L2

cos(1 2 )
L3 = 0
cos(3 1 )

or L2cos( ) = L3 cos ( )

Sol.
...(ii)

Similarly, the equation of the altitudes through B


and C are respectively
L3 cos(1 2) = L1 cos (2 3)
...(iii)
and L1cos(2 3) = L2 cos (3 1)

...(iv)

The orthocentre being the intersection of altitudes,


satisfies (ii), (iii) and (iv). So the orthocentre is
given by
L1cos(2 3) = L2 cos (3 1) = L3cos(1 2).
Ans.
Ex.46

p1 and p2 are the distances of a variable point from


the two lines 2x + y 1 = 0 & x + 2y + 1 = 0

Show that the straight lines given by


(i) x (a + 2b) + y (a + 3b) = a + b for different
values of a and b pass through a fixed point.

p12 p 22 = 3 (h + k) (h k 2) = 5
or h2 k2 = 5 + 2h + 2k
...(i)
Now product of perpendicular from (h, k) to other
two lines is d1d2 =

h k 2 h k 3
.
2
2

1
(h 2 k 2 ) (2h 2k ) 3(h k ) 6
2
=

1
[5 + 2h + 2k + 2h 2k 3h 3k 6]
2

h 3k 1
by (i)
2
Again if p be distance of (h, k) from
x 3y 1 = 0
=

then p =

h 3k 1

...(ii)

...(iii)

10

51

d1d 2

10
2

From (i),

(iii)
Hence d1. d2 varies as p.
Ex.48

Ans.

Prove that the product of the perpendiculars from


the points [

Sol.

or (ax1 + by1 + c) + (ax2 + by2 + c) = 0

= constant, by (ii) and

(a 2 b 2 )

(ax1 by1 c) (ax 2 by 2 c)


a 2 b2
2(ax1 by1 c)
=
, using (iii)
a 2 b2
2(ax1 by1 c)
x 2 x1 y 2 y1

=
a
b
a 2 b2

(a 2 b 2 ) ( b cos ) ab
( b 2 cos 2 a 2 sin 2 )

= b2
Ex.49

Ans.

(a 2 b 2 ) ( b cos ) ab

( b 2 cos 2 a 2 sin 2 )

p1p2 =
= b2

a
a

2
2

Ex.50

(a 2 b 2 ) b 2 cos 2 a 2 b 2
b 2 cos 2 a 2 sin 2

a 2 cos 2 b 2 cos 2
b 2 cos2 a 2 sin 2
sin 2 b 2 cos 2
2

b cos a sin

=b.
2

Sol.
Ans.

If x cos + y sin = p where p = sin2/cos be


a straight line, prove that perpendiculars, p 1, p2 &
p3 on this line from the points (m 2, 2m),
(mm', m + m') and (m' 2, 2m') respectively are in
geometric progression.
The given line is x cos + y sin = p.
Length of perpendiculars from (m2, 2m),
(mm',m +m') (m'2, 2m') are p1, p2, p3 respectively,
p1 =

If the image of the point (x1, y1) with respect to the


mirror ax + by + c = 0 be (x2, y2), show that

2(ax1 by1 c)
x 2 x1
y y1
= 2
=
.
a
b
a 2 b2
Sol.

a ( x1 x 2 ) b( y1 y 2 )
,
a 2 b2

x (b cos ) + y (a sin ) ab = 0

p2 =

x 1 x 2 y1 y 2

a
b

by ratio and proportion

, 0] to the line

x
y
cos +
sin = 1 is b2.
a
b
Equation of the given lines is

p1 =

...(iii)

m 2 cos 2m sin p
(cos 2 sin 2 )

= m2 cos + 2m sin +

Clearly, the line PQ joining the points P(x1, y1) and


Q(x2, y2) is perpendicular to the line
ax + by + c = 0; also the mid-point M of PQ is on
the lines ax + by + c = 0.

sin 2
cos

(m cos sin )2
.
cos

Similarly, p2 =

mm' cos (m m' ) sin p


(cos 2 sin 2 )

= mm' cos + (m + m') sin + sin2 /cos


= [mm' cos2+ (m + m') sin cos + sin2]/cos
= (m cos + sin) (m' cos + sin) /cos
and p3 = (m' cos + sin)2/cos, as in p1
p1p3 =

(m cos sin )2 (m' cos sin )2


=
cos2

p 22

y1 y 2 a
=1
x1 x 2 b

and a .

Hence p1, p2, p3 are in geometric progression.


Ans.

...(i)

x1 x 2
y y2
+ b. 1
+ c = 0 ...(ii)
2
2

From (ii), a(x1+ x2) + b (y1 + y2) + 2c = 0

Ex.51

A ray of light is sent along the line x 2y = 8.


After refracting across the line x + y = 1 it enters
the opposite side after turning by 15 away from

51

Sol.

the line x + y = 1. Find the equation of the line


along which the refracted ray travels.
The point of incidence A is the intersection of

5 3 3

=
=

95 3

(5 3 3)(9 5 3 )
81 75

30 3 48
=5
6

+ 8.

If the angle between x + y =1 and the line through


5 3 6
13

A (10/3, 7/3) with the slope

be

then
x 2y = 8
...(i)
x+y=1
...(ii)
Solving (i), (ii) we get
y = 7/ 3, x = 10 / 3 ;
A = (10 / 3, 7/ 3).
Let the refracted ray have the slope = m

1
2
1
1 m.
2
m

Then tan 15 =

or tan (45 30) =

2m 1
m2

or

1
3 1
3 1

tan

2m 1
m2

3
1 =

m =

)m=2

5 3 6 5 3 6
,
3
13

through A (10/3, 7/3) with the slope

5 3 6
1
3
5 3 6
1
3

19 (5 3 )

208 130 3
=
286

5 3

85 3
=
11

80 50 3
0
11

tan > tan ; >


the slope of the refracted ray =

5 36
3

y+
5 3 6
(1)
3
5 3 6
1
( 1)
3

5 3 6
3

the equation of the refracted ray is

be . Then
=

75 3
19 5 3

55 3 88 8 5 3
11

Let the angle between x + y = 1 and the line

tan

(7 5 3 )(19 5 3 )

As

3 ) (m + 2)

3 m = 5 2 3 , (4

75 3
19 5 3

2m 1
2 3 ,2 3
m2

2m 1 = (2

85 3
11

3
2

5 3 6
1
13
5 3 6
1
13

2m 1
m2

5 3 6
(1)
13
5 3 6
1
(1)
13

7
5 3 6
10
=
x

3
3
3

or 3(5 2 3 )x 3

3 y + 13

=
Ex.52

Sol.

50 = 0.
Ans.

Find the equation of the line which bisects the


obtuse angle between the lines x 2y + 4 = 0 and
4x 3y + 2 = 0.
The equations of the lines with the constant terms
of the same sign are
x 2y + 4 = 0
...(i)
and 4x 3y + 2 = 0
...(ii)

51

the equations of the bisectors of the angles


between the lines (i) and (ii) are
x 2y 4
4 x 3y 2
=
...(iii)
(1 4)
16 9
Taking +ve sign on R.H.S. of (iii), the bisectors of
the angle in which origin lies is
(4 5 ) x + (2 5 3) y (4 5 2) = 0
...(iv)
and taking ve sign on R.H.S. of (iii), the bisectors
of the anlge in which origin does not lie is
(4 + 5 ) x (2 5 + 3) y + (4 5 + 2) = 0
...(v)
Now let be the angle between the line (i) and the
bisector (iv), whose slopes are
m1 = 1/2 and m2 = (4
5 ) (2 5 3)
respectively, then

tan =

1
4 5

2 2 5 3
1 4 5
1 .
2 2 5 3

or 642 112 + 49 = 0 or (8 7) 2 = 0
= 7/8
Putting for in (i) and simplifying we get
the required line as 2x + y 5 = 0
(b) Put b/a = then ( 1)2 = 0
= 1, 3x y + 1 = 0
Since there is only value of hence there
will be only one line.
Ans.
Ex.54

P Q
{(p q ) 2 (q p ) 2 }

The denominator on both sides cancel as each is

5 2

{( p 2 q 2 )(1 ) 2 }

Hence the bisectors are P + Q = (P Q).


or P = 0 or Q = 0
px + qy + r = 0 or qx py + r' = 0
Clearly these lines are perpendicular as we know
that the bisectors of two given lines are always
perpendicular.
Ans.

( 5 2)
( 5 2)( 5 2)

Hence the bisectors (iv) is the bisector of the


obtuse angle between the given lines. Ans.

Ex.55

(a) Find the equations of the lines through the


point of intersection of the lines
x 3y + 1 = 0 and 2x + 5y 9 = 0 and whose
distance from the origin is 5 .
(b) Given the pencil of lines
a(2x + y + 4) + b (x 2y 3) = 0. Prove
that among the line of the pencil there
exists only one line whose distance from

Sol.

and

px + qy + r (qx py + r') = 0 are two given


lines. Determine the equations of their bisectors.
Sol.
The required bisectors are
P Q
=

{(p q) 2 (q p ) 2 }

= ( 5 + 2) > 1
> 45 2 > 90.

Ex.53

px + qy + r +(qx py + r' = 0

the point (2, 3) is 10 .


Write the equation of this line.
(a) The required line by P + Q = 0 is

Sol.

If u = a1x + b1y + c1 = 0
and v = a2x + b2y + c2 = 0,
and a1/a2 = b1/b2 = c1/c2,
prove that the curve u + kv = 0 is nothing but any
of the given straight lines u = 0 or v = 0
The given lines are
u = a1x + b1y + c1 = 0
...(i)
and v = a2x + b2y + c2 = 0.
...(ii)
Also given that

a 1 b 1 c1

= (say)
a 2 b2 c2
a1 = a2, b1 = b2, c1 = c2

(x 3y + 1) + (2x + 5y 9) = 0

Now u = a1x + b1y + c1 = (a2x + b2y + c2) = v

or x(1 + 2) + y(5 3) + (1 9) = 0 ...(i)

Line u = 0 becomes v = 0 or v = 0

Its distance from origin is given to be

If condition (iii) is satisfied, then u = 0 and


v = 0 represent the same straight line.
Also the curve u + kv = 0 is v + kv = 0

1 9
[(1 2) (5 3) 2 ]1 / 2
2

5 ,

Square
(1 9)2 = 5(1 + 42 + 4+ 252 30 + 9)

or (+ k) v = 0 or v = 0

51

Hence the curve u + kv = 0 is nothing but any of


the given straight lines u = 0 or v = 0.
Ans.
Ex.56

Sol.

Find the equation to the pair of lines through the


origin which are perpendicular to the lines
represented by ax2 + 2hxy + by2 = 0.
If the lines represented by ax2 + 2hxy + by2 = 0 be
y m1x = 0 and y m2 x = 0 then
2h
a
m1 + m2 =
and m1 m2 =
b
b
The lines perpendicular to them and passing
through origin will be
1
1
y=
x and y =
x
m1
m2
Their combined equation is
(m1y + x) (m2y + x) = 0
or m1m2y2 + xy (m1 + m2) + x2 = 0
or

{(0, 0) being a point on 3x + 4y = 0}


=

d' =

Sol.

1
5

1
32 42

1
5

d = d'

Hence, the lines form a square.

Ans.

Ex.58

For what value of does the equation

Sol.

12x2 10xy + 2y2 + 11x 5y + = 0 represent a


pair of straight lines. Find their equations and the
angle between them.
Comparing the given equation, with the equation
ax2 + 2hxy + by2 + 2gx + 2fy + c = 0
we get
a = 12, h = 5, b = 2, g = 11/2, f = 5/2, c =

2h

a 2
+ x2 = 0
y + xy
b
b

Show that the four lines given by


12x2 + 7xy 12y2 = 0 and 12x2 + 7xy 12y2 x
+ 7y 1 = 0 lie along sides of a square.
12x2 + 7xy 12y2 = 12x2 + 16xy 9xy 12y2
= 4x (3x + 4y) 3y (3x + 4y)
= (3x + 4y) (4x 3y)
The pair of lines given by the first equation has
their separate equations 3x + 4y = 0
4x 3y = 0 which are at right angle.
Again 12x2 + 7xy 12y2 x + 7y 1
= (3x + 4y) (4x 3y) x + 7y 1
(factorized as above)
= (3x + 4y 1) (4x 3y + 1), by trial
The pair of lines given by the second equation has
their separate equations 3x + 4y 1 = 0 and 4x
3y + 1= 0 which are at right angle.
Thus the four line are 3x + 4y = 0, 4x 3y = 0,
3x + 4y 1= 0 and 4x 3y + 1 = 0 of which the
first and the third are parallel while the second and
the fourth are also parallel. Moreover the first and
the second are at right angle. Hence, the lines form
a rectangle.
The distance between 3x + 4y = 0 and
3x + 4y 1 = 0 is given by
d = distance of (0, 0) from 3x + 4y 1 = 0

3 4

The distance between 4x 3y = 0 and


4x 3y + 1 = 0 is given by

If the given equation represents a pair of st. lines,


then abc + 2fgh af 2 bg2 ch2 = 0

or bx2 2hxy + ay2 = 0


Rule : Interchange the coefficients of x 2 and y2 and
change the sign of the term of xy. Ans.
Ex.57

1
2

or 12.2. + 2 (5/2). (11/2). (5) 12.(5/2) 2 2.


(11/2) 2 . (5) 2 = 0
= 2. Also h2 ab = 25 24 > 0.
The given equation will represent a pair of st.
line if = 2.
To find the two lines. Substituting = 2, in the
given equation we get
12x2 10xy + 2y2 + 11x 5y + 2 = 0
...(i)
2
2
Since 12x 10xy + 2y = 2 (3x y) (2x y),
factors of (i) can be taken as,
2(3x y + a) (2x y + b) so that
12x2 10xy + 2y2 + 11x 5y + 2
2(3x y + a) (2x y + b)
12x2 10xy + 2y2 + 2(3b + 2a)x
2(a + b) y + 2ab
Comparing, 2a + 3b = 11/2, a + b = 5/2, ab = 1
Solving, we get a = 2, b = 1/2
Thus the factors of (i) are
2(3x y + 2) (2x y + 1/2) = 0
or (3x y + 2) (4x 2y + 1) = 0
The two st. lines represented by the given
equation are
3x y + 2 = 0 and 4x 2y + 1 = 0
Ans.
Ex.59

If the equation ax2 + 2hxy + by2 + 2gx + 2fy + c = 0


represents two straight line prove that the product

51

m1 = tan and m2 = tan

of the perpendiculars drawn from the origin to the


c

lines is
Sol.

(tan tan )2
= (m1 m2)2 = (m1 + m2)2 4m1m2

(a b ) 2 4 h 2

Let ax2 + 12hxy + by2 + 2gx + 2fy + c


(1x + m1y + n1) (2x + m2y + n2).
Then the separate lines are
1x + m1y + n1 = 0 and 2x + m2y + n2 = 0
where 12 = a1, m1m2 = b, n1n2 = c,
1m2 + 2m1 = 2h, m1n2 + m2n1 = 2f,
n12 + n21 = 2g
Lengths of perpendiculars from (0, 0) to the lines
are respectively

n1
2

=
=
=
Ex.60

n1
2
1

Ex.61

m2

(a b ) 4 h

sin cos
4
sin 2 cos 2

[1 sin2 cos
(cos2 cos4)

4 cos 2 (1 cos 2 )
sin 2 cos 2

=4

Prove that the general equation ax2 + 2hxy + by2 +


2gx + 2fy + c = 0 will represent two parallel
straight lines if g2 ac 0 and
ab , g

b =f

a )

a
h
g
=
=
h
b
f
The given equation
ax2 + 2hxy + by2 + 2gx + 2fy + c = 0
...(i)
will represent a pair of parallel lines if
(1) abc + 2fgh af 2 bg2 ch2 = 0
...(ii)
Also prove that

a b (2h ) 2 2ab
2

[1 cos2(tan2 + cos2)]

(g 2 ac)
.
a (a b)

sin cos 2
4

a b (1m 2 2 m1 ) 212 . m1m 2


2

4
2

4(tan 2 cos 2 )
sin 2

or (h = ab , g b = f a )].
Also prove that the distance between them is

12 22 m12 m 2 12 m 22 22 m12
2

sin 2 cos 2

[(h =

n1n 2

Hence tan tan = 2. Ans.

n2
2
2

Sol.

Ans.

Show that the difference of the tangents of the


angles which the lines (tan2 + cos2 )x2
2xy tan + sin y = 0 make with the axis of x
is 2.
The given equation is
(tan2 + cos2 x2 2xy tan + sin2 y2 = 0
..(i)
2
2
a = tan + cos , h = tan , b = sin2
If the lines represented by (i) are
y m1x = 0 and y m2x = 0, then
2

Sol.

2 m 2

2
m1

The required product of perpendiculars drawn


from the origin
=

n2

and

1 m 1

2
2 tan
m1 + m2 =
=
=
,
2
sin cos
b
sin
2h

a
tan 2 cos 2
=
b
sin 2
Now, if the two lines make angles and
respectively to the axis of x, then

and m1m2 =

2
and (2) tan = 2 h ab = 0 i.e.h2 = ab ...(iii)
ab
From (iii) taking h = (ab), we have from (ii)
abc + 2fg ab af 2 bg2 c. ab

= [f a g b ]2 = 0 if g
Thus one set of conditions is
h = ab , g b = f a
Again using h =
abc 2fg.

b =f

...(iv)

ab , we have from (ii)

ab af 2 bg2 c.ab

= [f a +g b ]2 = 0 if g b = f
Thus the second set of condition is
h = ab ; g b = f a
Now from (iv)

51

...(v)

a
=
h

ab

g
=
f

and

a h
,
=
b b

a
b

ab
=
b

a
.
b

a
h
g
=
=
h
b
f
Also from (v), we have in the similar manner that
a
h
g
=
=
h
b
f
Thus in either case, we have
a
h
g
=
=
...(vi)
h
b
f
The distance between the parallel lines
Using h = ab in (i), we have

ax2 + 2

ab . xy + by2 + 2gx + 2fy + c = 0

or (x a + y b )2 + 2gx + 2fy + c = 0 ...(vii)


Let (vii) be equivalent to
(x a + y b + p) (x a + y b + q) = 0
or ax2 + 2

ab . xy + by2 + (p + q)

a .x

+ (p + q) b . y + pq = 0
Comparing (vii) and (viii), we get
p + q = 2g/ a = 2f/ b , pq = c

...(viii)
...(ix)

Distance between the parallel lines


=

|pq|
=
(a b )

4g 2

a 4c

(p q ) 2 4pq

ab

(a b)

g 2 ac

a (a b)

= 2

Since distance of two parallel lines is real so


g2 ac 0
This distance can also be proved to be equal to
f 2 bc
.
b(a b)

Ans.

51

EXERCISE # 1
Q.1

Q.8

The line segment joining the points (1, 2) and


(2, 1) is divided by the line 3x + 4y = 7 in the
ratio (A) 3 : 4
(B) 4 : 3
(C) 9 : 4
(D) 4 : 9

Q.9

The line segment joining the points (3, 4) and


(1, 2) is divided by y-axis in the ratio
(A) 1 : 3
(B) 2 : 3
(C) 3 : 1
(D) 3 : 2

Q.10

If m1 and m2 are roots of the equation

The cartesian coordinates of the points whose

polar coordinates are 5, equal to 4

(A)

(C)

Q.2

,
,

(B)

5 5

,
2
2

(D)

The polar form of the equation x2 + y2 = ax is (A) r = a sin


(B) r = a cos
(C) r = a sin

(D) None of these

x2 + (

Q.3

(A)

a b p q

(B)

a b 2p q

(C) 2
(D)

1
2

1) = 0 then the area of

(C)

33
4

11

c 2 (B)

33 11 2
c (D)

33 11 2
c

33
2

11

c2

a 2 b2 p2 q 2
a 2 b2 p2 q 2

The distance between point (2, 15) & (1, 75) is (A) 1
(B) 3
(C) 2 3 (D) 3

Q.5

The coordinates of base BC of an isosceles


triangle ABC are given by B (1, 3) and C (2, 7)
which of the following points can be the possible
coordinates of the vertex A.
(A) (7, 1/8)
(B) (1, 6)
(C) (1/2, 5)
(D) (5/6, 6)

Q.7

(A)

Q.4

Q.6

+ 2) x + (

the triangle formed by the lines y = m 1x, y = m2x


and y = c is -

The abscissa of two points A, B are the roots


of the equation x2 + 2ax b2 = 0 and their ordinate
are the roots of x2 + 2px q2 = 0 then the distance AB
in terms of a, b, p, q is 2

If vertices of a quadrilateral are A(0, 0), B(3, 4), C(7,


7) and D (4, 3), then quadrilateral ABCD is
(A) parallelogram
(B) rectangle
(C) square
(D) rhombus

P and Q are points on the line joining A( 2, 5) and


B(3, 1) such that AP = PQ = QB then the mid point
of PQ is (A) (1/2, 3)
(B) ( 1/2, 4)
(C) (2, 3)
(D) (1, 4)

Q.11

The points (0, 1), (2, 3), (6, 7) and (8, 3) are(A) Collinear
(B) Vertices of parallelogram which is not a
rectangle
(C) Vertices of rectangle which is not a square
(D) None of these

Q.12

The area of the pentagon whose vertices are


(4, 1), (3, 6), (5, 1), (3, 3) and (3, 0) is(A) 30 unit2
(B) 60 unit2
(C) 120 unit2

(D) none of these

Q.13

If the vertices of a triangle be (0, 0), (6, 0) and


(6, 8), then its incentre will be(A) (2, 1) (B) (1, 2) (C) (4, 2) (D) (2, 4)

Q.14

The coordinates of the middle points of the sides


of a triangle are (4, 2), (3, 3) and (2, 2) then the
coordinates of its centroid are-

(A) 3,
3

(C) (4, 3)
Q.15

(B) (3, 3)
(D) None of these

The orthocentre of the triangle formed by the lines


4x 7y +10 = 0, x + y = 5 and 7x + 4y = 15, is-

(A) (1, 2)
(C) (1, 2)

(B) (1, 2)
(D) (1, 2)

1 3

,
2
2

(A)

(C)
Q.16

The locus of the points of intersection of the lines


x cos + y sin = a and x sin y cos = b (where
is a variable) is
(A) x2 + y2 = a2 + b2
(C) x2 + y2 = a2 b2

Q.17

Q.18

Q.19

Q.20

Q.21

Q.22

4
P2

Q.23

(D)

If P = (1, 0) and Q = (1, 0) and R = (2, 0) are


three given points, the locus of the point S
satisfying the relation SQ2 + SR2 = 2SP2 is(A) a straight line parallel to the x-axis
(B) a circle passing through the origin
(C) a circle with the centre at the origin
(D) a straight line parallel to the y-axis

Reflecting the point (2, 1) about Y axis


coordinate axis are rotated at 45 angle in negative
direction without shifting the origin . The new
coordinates of the point are-

(D) None of these

If the axes are rotated through an angle of 30 in

(A) (2,

3 )

(C) ( 3 , 2)

3 ) in

(B) ( 3 , 5)
(D) (2, 3)

Q.24

The equation of a line which makes an angle of


tan1 (3) with the x-axis anticlockwise & cuts off
an intercept of 4 units on negative direction of yaxis is (A) y = 3x + 4
(B) y = 3x 4
(C) x = 3y + 4
(D) None of these

Q.25

A line passes through (x1, y1). This point bisects


the segment of the line between the axes. Its
equation is-

Q.26

On shifting the origin to the point (2, 5) and


keeping the axis parallel the new coordinates of
the point (5, 3) will be(A) (3, 2)
(B) (3, 2)
(C) (7, 8)
(D) None of these
At what point the origin be shifted, if the
coordinates of point (4, 5) becomes (3, 9)
(A) (7, 4)
(B) (7, 4)
(C) (7, 4)
(D) None of these

the new system was formerly

1
1
2
2 2
2
x
y
P

The locus of a point which moves so that the


algebraic sum of the perpendiculars let fall from it
on two given straight lines is constant, is(A) a circle
(B) a straight line
(C) a pair of lines
(D) none of these

, 2

the clockwise direction, the point (4, 2

(B) x2 y2 = a2 + b2
(D) None of these

The locus of the mid point of the portion intercept


between the axes by the line x cos + y sin = P
where P is a constant is1
1
4
(A) x2 + y2 = 4P2
(B) x 2 y 2 P 2
(C) x2 + y2 =

(B)

Q.27

(A)

x
y

2
x1 y1

(B)

x
y 1

x1 y1 2

(C)

x
y

1
x1 y1

(D) None

The equation of the straight line on which the


length of the perpendicular from the origin is 2
and the perpendicular makes an angle with
x-axis such that sin =

1
is 3

(A) 2

2 xy=6

(B) 2

2 x+y=6

(C) 3

2 x+y=6

(D) 2

2 xy=5

If the straight line through the point P (3, 4) makes


an angle

with x-axis and meets the line 12x +


6

5y + 10 = 0 at Q. Then the length of PQ is 132


132
(A)
(B)
12 3 5
12 3 5
(C)

132
3 5

(D) None of these

Q.28

One side of square is x y = 0 find side opposite


to it if length of square is 2 2
(A) x y = 4
(B) x + y = 4
(C) x + y = 4
(D) x y = 4

Q.29

Q.36

x y
0
b a

(C)

Q.31

Q.32

1 3
(C) ,
5 5

Q.35

Q.38

The image of the point (2, 1) with respect to the


line mirror be (5, 2). Then the equation of the
mirror is (A) 3x + y 12 = 0
(B) 3x y + 12 = 0
(C) 3x + y + 12 = 0
(D) 3x y 12 = 0
Perpendicular bisector of segment PQ is
3x + 4y 2 = 0. If P is (1, 1) then point Q is -

1 3
,
5 5

Q.34

Q.37

x y b

b a a

The equation of the perpendicular bisector of the line


segment joining points (1, 5) and (3, 2) is (A) 4x + 3y 29 = 0 (B) 4x + 3y 13 = 0
(C) 8x + 6y 13 = 0 (D) 8x + 6y + 13 = 0

(A)

Q.33

(D)

1 3

(B) ,
5 5

1 3
(D) ,
5 5

The coordinates of a point on x + y + 3 = 0, whose


distance from x + 2y + 2 = 0 is 5 is equal to (A) (9, 6)
(B) (9, 6)
(C) (9, 6)
(D) None of these

The equation of the line through the point of


intersection of the lines 2x + 3y 7 = 0 and
3x + 2y 8 = 0 which cuts equal intercepts on the
axes is (A) x + y = 3
(B) 2x + 2y = 7
(C) x + y = 1
(D) 3x + 3y = 8
The point of intersection of the lines

The equation of the bisector of the acute angle


between the lines 3x 4y + 7 = 0 and
12x + 5y 2 = 0 is (A) 21x + 77y 101 = 0
(B) 11x 3y + 9 = 0
(C) 11x 3y 9 = 0
(D) none of these
On the portion of the straight line x + y 7 = 0
which is intercepted between the axes a square is
constructed on the side of the line away from the
origin. Then the equation to the diagonals are (A) x + y = 7 ; x y = 7
(B) x = 7 ; y = 7
(C) x = 7 ; y = 7
(D) x = 7 ; y = 7

Q.39

The equation of the line through the point of


intersection of the lines x y + 4 = 0 and
y 2x 5 = 0 and passing through the point (3, 2) is
(A) x 4y + 5 = 0
(B) x + 4y 11 = 0
(C) 2x y 4 = 0
(D) none of these

Q.40

The equation of the line passing through the


intersection of x

3 y +

3 1 = 0 and

x + y 2 = 0 and making an angle of 15 with the


first line is
(A) x y = 0
(B) x y + 1 = 0
(C) y = 1

(D)

x y +1

=0

Q.41

If a + b + c = 0 then the straight line


2ax + 3by + 4c = 0 passes through the fixed point(A) (2, 4/3)
(B) (2,2)
(C) (4/3, 4/2)
(D) none of these

Q.42

Find the separate equations of the straight lines


whose joint equations is
ab (x2 y2) + (a2 b2) xy = 0
(A) bx + ay = 0 and ax by = 0

x y
1
a b

x y
1 does not lies on the line b a
(A) x y = 0
(B) (x + y) (a + b) = 2ab
and

For what value of , the three lines


2x 5y + 3 = 0, 5x 9y + = 0 & x 2y + 1 = 0,
are concurrent (A) 4
(B) 5
(C) 3
(D) 2

The equation of a line perpendicular to the line

x y
= 1 and passing through the point where it
a
b
meets x-axis is x y a
x y a
(A) 0
(B)
b a b
a b b

Q.30

(C) (x + my) (a + b) = ( + m) ab.


(D) (x my) (a b) = (1 m) ab

(B) bx ay = 0 and ax + by = 0
(C) bx ay = 0 and ax by = 0
(D) None of these

Q.54

Q.43

The equation ax2 + by2 + c (x + y) = 0 represents a


pair of straight lines if (A) c = 0
(B) a + b = 0
(C) Both (A) & (B)
(D) none of these

Q.44

If lines px2 qxy y2 = 0 make angle '' and ''


with x-axis then value of tan(+ ) is (A)

q
1 p

p
1 q

(B)
(D)

q
1 p

(C)

p
1 q

Q.45

Only in case of acute angled triangle centroid


divides line joining circumcentre and orthocentre
in the ratio 2 : 1

Q.46

If L1 = 0 and L2 = 0 are parallel lines, then family


of lines will be L1 + L2 = 0

Q.47

For a triangle there exists a unique point whose


distance from all three sides is same and it is
called incentre of triangle

Q.48

If a and b are real numbers between 0 and 1, such


that the points (a, 1) (1, b) and (0, 0) form an
equilateral triangle then 2(a + b) ab is equal
to ..............

Q.49

The no. of points (p, q) such that p, q {1, 2, 3, 4}


and the equation px2 + qx + 1 = 0 has real roots
is............

Q.50

If , , are the real roots of the equation


x3 3px2 + 3qx 1 = 0 then centroid of the
triangle with vertices (, 1/), (, 1/) and
(, 1/) is at the point ............

Q.51

The integral values of for which origin lies in


the bisector of acute angle between lines
(2 + 3) x + 4y + 3 = 0 and x + y + 1 = 0
is ..............

Q.52

In a triangle if vertex A is (2, 3) and angle bisector


through B is x + 2y = 3 and median through C is x
2y = 1, then co-ordinate of vertex B
is...............

Q.53

The distance between lines whose combined equation


are x2 + 2 2 xy + 2y2 + 4x + 4 2 y + 1 = 0 is

The slopes of two lines represented by


x2 (tan2 + cos2) 2xy tan + y2 sin2 = 0 are m1
and m2, then |m1 m2| is equal to.......

EXERCISE # 2
4 3 1
,
3
3

(C)
Q.1

If one vertex of an equilateral triangle of side 'a'


lies at the origin and the other lies on the line

Q.8

Q.2

(A) (0, a)

(B)( 3 a/2, a/2)

(C) (0, a)

(D) All of these

Let P = (1, 1) and Q = (3, 2). The point R on the xaxis such that PR + RQ is the minimum is
5

, 0
3

(C) (3, 0)
Q.3

Q.4

If coordinates of orthocentre and centroid of a


triangle are (4, 1) and (2, 1), then coordinates of a
point which is equidistant from the vertices of the
triangle is (A) (2, 2)
(B) (3, 2)
(C) (2, 3)
(D) (1, 2)

Q.10

One vertex of the equilateral triangle with


circumcentre at (1, 1) and one side as x + y = 3 is(A) (2, 2)
(B) (0, 0)
(C) (2, 2)
(D) None

Q. 11

A point moves such that its distance from the point


(4, 0) is half that of its distance from the line x =
16. The locus of this point is
(A) 3x2 + 4y2 = 192
(B) 4x2 + 3y2 = 192
(C) x2 + y2 = 192
(D) None of these

Q. 12

Let A = (1, 0) and B = (2, 1). The line AB turn

(D) none

If the line segment joining (2, 3) and (1, 2) is


divided internally in the ratio 3 : 4 by the line
x + 2y = k then k is
(A)

41
7

36
(C)
7
Q.5

Q.9

(B)

The four points whose coordinates are (2,1), (1,4),


(4,5), (5,2) form :
(A) a rectangle which is not a square
(B) a trapezium which is not a parallelogram
(C) a square
(D) a rhombus which is not a square

(B)

5
7

31
(D)
7

about A through an angle /6 in the clockwise


sense, and the new position of B is B. Then B has
the coordinates

Let A = (1, 2), B = (3, 4) and let C = (x, y) be a


point such that (x1) (x3) + (y 2) (y 4) = 0.

3 3 3 1

,
2
2

(A)

If ar (ABC) = 1 then maximum number of


positions of C in the x-y plane is
(A) 2
(B) 4
(C) 8
Q.6

Q.7

(A) 71 / 2 sq. units

(B) 142 / 2 sq. units

(C) 2

(D)

142 sq. units

In the ABC, the coordinates of B are (0, 0)


AB = 2, ABC = /3 and the middle point of BC
has the coordinates (2, 0). The centroid of the
triangle is
1
3
(A) ,

2 2

5 1

(B) ,
3 3

(B)

3 3 3 1

2
2

(D) none

The vertex O of an isosceles triangle OAB lies at


the origin and the equation of the base AB is
x y + 1 = 0. If OA = OB = 6, the area of the
triangle OAB
71 sq. units

3 y + x = 0 Then the following is an

interior point of the triangle (A) Circumcentre


(B) Centroid
(C) Orthocentre
(D) None of these

, 0
3

(A)

The sides of a triangle are x + y = 1, 7y = x


and

x 3 y = 0, the co-ordinates of the third vertex are

(D) none of these

1 3 1 3

,
2
2

(C)

(D)

none

of

these
Q. 13

The point (4, 1) undergoes the following three


transformation successively :
(i) Reflection about the line y = x
(ii) Transformation through a distance 2 units
along the positive direction of x-axis
(iii) Rotation through angle /4 about the origin in
the anticlockwise direction. The final position
of the point is given by the coordinates :
7
1

,
(A)
(B) (2,7 2 )
2
2


1
7

,
(C)
2
2

Q. 14

The image of the point A (1, 2) by the mirror


y = x is the point B and the image of B by the line
mirror y = 0 is the point (, ). Then(A) = 1, = 2
(B) = 0, = 0
(C) = 2, = 1
(D) none of these

Q.15

The distance of the line x + y 8 = 0 from (4, 1)


measured along the direction whose slope is 2 is
(A) 3 5 (B) 6 5 (C) 2 5 (D) None

Q.16

In what direction a line be drawn through the point


(1, 2) so that its point of intersection with the line
x + y = 4 is at a distance

Q.17

0, 4 5 3

or

(D) 0,
2

Q.22

If

cos
sin

1,
1 and
3
2

3x 2y + 1 = 0, then belongs to the interval

(A) ,

3

,

4
4

(C)
Q.23

0, 4 5 3

(B) [ , ]

(D) none of these

Given four lines whose equations are


x + 2y 3 = 0, 2x + 3y 4 = 0, 3x + 4y 7 = 0
and 4x + 5y 6 = 0 then
(A) they are all concurrent

4 5 3
(C) 0,

(D) none of these

(C) They are sides of trapezium

Q.24

If a, b, c are in A.P., then ax + by + c = 0


represents (A) a single line
(B) a family of concurrent lines
(C) a family of parallel lines
(D) none of these

Q.25

If the lines represented by x 2 2pxy y2 = 0 are


rotated about the origin through an angle one in
clockwise direction and other in anti-clockwise
direction, then the equation of the bisectors of the
angle between the lines in the new position is
(A) px2 + 2xy py2 = 0
(B) px2 + 2xy + py2 = 0
(C) x2 2pxy + y2 = 0
(D) None of these

Q.26

If the equation 12x2 +7xy py2 18x + qy + 6 = 0


represent a pair of perpendicular straight line then
(A) p = 12, q = 1
(B) p = 1, q = 12
(C) p = 1, q = 12
(D) p = 1, q = 12

5 5 3
,
(D)
2
2
If the vertices of a quadrilateral is given by
(x2 4)2 + (y2 9) 2 = 0 then area of quadrilatural
is(A) 36
(B) 24
(C) 16
(D) 81

The equation of the line through the point


(5, 4) such that its segment intercepted by the
lines x + 2y + 1 = 0 and x + 2y 1 = 0 is of length
2
5

1),

line

B(1,

are two points on the same side of the

(B) they are sides of a quadrilateral

is -

(A) 2x y = 4
(C) 2x y = 0
Q.20

(C) (0, )

4 5 3
(B) 0,

Q.19

(B) ,
2 2

The straight line y = x 2 rotates about a point


where it cuts x-axis and becomes perpendicular on
the straight line ax + by + c = 0 then its equation is
(A) ax + by + 2a = 0 (B) ay bx + 2b = 0
(C) ax + by + 2b = 0 (D) none of these

depending on which the point P is takes.

Q.18

(A)

Q.21

6
from the given
3

point (A) 75
(B) 60
(C) 45
(D) 30
P is a point on either of the two lines
y 3 |x| = 2 at a distance of 5 units from
their point of intersection. The coordinates of
the foot of the perpendicular from P on the
bisector of the angle between them are (A)

3
,
.
2 2

(D) ( 2 ,7 2 )

(B) 2x y = 14
(D) none

If the point (cos, sin) does not fall in that angle


between the lines y = | x 1 | in which the origin
lies then belongs to -

Q.27

Q.28

Q.29

Q.30

One of the bisectors of the angle between the lines a


(x 1)2 + 2h (x 1) (y 2) + b (y 2) 2 = 0 is
x + 2y 5 = 0 The other bisector is (A) 2x y = 0
(B) 2x + y = 0
(C) 2x + y 4 = 0
(D) x 2y + 3 = 0
If the two pairs of lines x2 2mxy y2 = 0 and
x2 2nxy y2 = 0 are such that one of them
represents the bisectors of the angles between the
other, then (A) mn + 1 = 0
(B) mn 1 = 0
(C) 1/m + 1/n = 0
(D) 1/m 1/n = 0
The number of values of for which bisectors of
the angle between the lines
ax2 + 2hxy + by2 + (x2 + y2) = 0 are the same is (A) two
(B) one
(C) zero (D) infinite
If the slope of one line is double the slope of
another line and the combined equation of the pair
2

of lines is

Q.31

Q.32

(C) (x 7y)2 + 14(x 7y) 15 = 0


(D) none of these
Q. 35

If P is a point which is at a distance of 4 units and


3 units from x-axis and y-axis respectively then
co-ordinate of P may be (A) (3, 4)
(B) (3, 4)
(C) (3, 4)
(D) (3, 4)

Q. 36

Let x(y 3) = 5 where x, y Integers then value


of x + y is equal to(A) 6
(B) 9
(C) 6
(D) 3

Q. 37

If P is a point on the line joining points A (2, 3) &


B (4, 5) such that AP =

Q.38

The number of lines passing through (2, 3) each


having distance equal to 5 units from the point
(7, 8) is :
(A) two
(B) zero
(C) one
(D) infinite

Q.33

The line 3x + 2y = 24 meets the y-axis at A and the


x-axis at B. C is a point on the perpendicular bisector
of AB such that the area of the triangle ABC is 91
sq. units. The coordinates of C are
(A) (29/2, 1)
(B) (29/2, 13)
(C) (13/2, 3/2)
(D) (13/2, 13)

Q.34

Three vertices of a quadrilateral in order are


(6, 1), (7, 2) and (1, 0). If the area of the
quadrilateral is 4 unit2 then the locus of the fourth
vertex has the equation
(A) x 7y = 1
(B) x 7y + 15 = 0

are(A) (3, 4)
(B) (2, 4)
(C) (1, 2)
(D) (3, 2)
Let A (2, ), B (3, 5), C (4, 5) are the vertices of
ABC whose area is 10(units)2, then value of

x
2xy y

0 then ab : h2 is
a
h
b

equal to (A) 9 : 8
(B) 3 : 2
(C) 8 : 3
(D) none of these
The four sides of a quadrilateral are given by the
equation xy (x 2) (y 3) = 0. The equation of the
line parallel to x 4y = 0 that divides the
quadrilateral in two equal areas is (A) x 4y + 5 = 0
(B) x 4y 5 = 0
(C) 4y = x + 1
(D) 4y + 1 = x

2 then co-ordinates of P

is/are (A) 20
(C) 20
Q.39

(B) 25
(D) 15

If area of OPB = area of OPA when O is origin,


A (6, 0), B (0, 4) and P lies on line
x + y = 1 then possible co-ordinate of P is/are-

3 2
,
5 5

Q.40

(A)

(B) (3, 2)

(C) (2, 1)

(D)

1 1
,
2 2

The combined equation of two sides of an


equilateral triangle is x2 3y2 2x + 1 = 0. If the
length of a side of the triangle is 4 then the
equation of the third side is (A) x = 2 3 + 1

(B) y = 2 3 + 1

(C) x + 2 3 = 1

(D) x = 2 3

Q.41

Two pairs of straight lines have the equations


y2 + xy 12x2 = 0 and ax2 + 2hxy + by2 = 0. One
line will be common among them if(A) a = 3 (2h + 3b) (B) a = 8 (h 2b)
(C) a = 2 (b + h)
(D) a = 3 (b + h)

Q.42

The diagonals of a square are along the pair of


lines whose equation is 2x2 3xy 2y2 = 0. If
(2, 1) is a vertex of the square then another vertex
consecutive to it can be (A) (1, 2)
(B) (1, 4)
(C) (1, 2)
(D) (1, 4)

Q.43

The pairs of straight lines ax2 + 2hxy ay2 = 0 and


hx2 2axy hy2 = 0 are such that(A) one pair bisects the angles between the other
pair
(B) the lines of one pair are equally inclined to the
lines of the other pair
(C) the lines of one pair are perpendicular to the
lines of the other pair
(D) none of these

The following questions 44 to 47 consists of


two statements each, printed as Assertion
and Reason. While answering these
questions you are to choose any one of the
following four responses.
(A) If both Assertion and Reason are true
and the Reason is correct explanation of
the Assertion.
(B) If both Assertion and Reason are true
but Reason is not correct explanation of
the Assertion.
(C) If Assertion is true but the

Reason (R) : Locus of points which is at equal


distance from the two given lines is the angle
bisectors of the two lines.

Q.48

Observe the following columns. Points given in


the column -I are collinear then
Column-I
Column -II
(A) (a, b + c), (b, c + a),
(P) if ad = bc
(c, a + b)
1
1
(B) (a, b), (c, d),(a + c, b + d) (Q) if
+
=1
a
b
(C) (a, 0), (0, b), (1, 1)
(R) if a = 1/2 , 1
(D) (a, 2 2a), (a + 1,, 2a) (S) always
( 4 a,6 a)

Q.49

Let P(x, y) be any point on the locus then observe


the following column
Column-I
Column -II
(A) The sum of the squares (P) x2 + y2 = 25
of distance from the
coordinate axis is 25
(B) distances to the coordinate (Q) 4x2 9y2 = 0
axes are in the ratio 2 : 3
respectively
(C) The square of whose (R) x2 + y2 = 4y
distance from origin is
4 time its y-coordinate
(D) Distance from P to (4, 0) (S) x2 3y2 8x
is double the distance
+ 16 = 0
form P to the x-axis
(T) 9x2 + 4y2 = 0

Q.50

The values of a,
Column-I
Column-II
(A) If (0, a) lies on or
(P) (4, 3]
inside the triangle formed by
the lines y + 3x + 2 = 0,
3y 2x 5 = 0,
4y + x 14 = 0
(B) If (2a 5, a2) is
(Q)(3,0)(1/3,1)
on the same side of
the x + y = 3 as that
of origin
(C) If (a, 2) lies between
(R) (5/3, 7/2)
the lines x y = 1 and
2(x y) + 5 = 0
(D) Point (a2, a + 1) lies
(S) (5/2, 3)
between the angles of
the lines 3x y + 1 = 0
and x + 2y 5 = 0 which
contains origin if

Q.51

The equation of the line through the intersection of


the line 2x 3y = 0 and 4x 5y = 2 and

Reason is false.
(D) If Assertion is false but Reason is true
Q. 44

Assertion (A) : In case of Isosceles triangle


circum centre, centroid, orthocentre, Incentre are
collinear.
Reason (R) : In case of Isosceles triangle ABC
where (AB = AC) perpendicular drawn from A
will bisect A, and will be perpendicular bisector
of side BC

Q.45

Assertion (A) : A point 'P' moves such that sum of


its distances from co-ordinates axis is 2 then area
of region generated by movement of 'P' is 8 sq.
units.
Reason (R) : Distance of point (h, k) from
x-axis is 'k' and from y-axis is 'h'.

Q.46

Assertion (A) : Image of orthocentre about any


sides of a always lies on the circumcircle of this
triangle.
Reason (R) : Circumcentre is always equidistant
from vertices of triangle.

Q.47

Assertion (A) : Each point on the line


y x + 12 = 0 is at same distance from the lines 3x
+ 4y 12 = 0 and 4x + 3y 12 = 0.

Column-I
Column-II
(A) Through the point
(P) 2x y = 4
(2, 1)
(B) to line x+ 2y + 1 = 0 (Q) x + y 5 = 0,
xy1=0
(C) || to line
(R) x y 1 = 0
3x 4y + 5 = 0
(D) Equally inclined to
(S) 3x 4y 1 = 0
axes
Q.52

Find the value of if the family of straight lines


(2x + 3y + 4) + (6x y + 12) = 0 is
Column-I
Column-II
(A) || to y-axis
(P) = 3/4
(B) to 7x + y 4 = 0

(Q) = 1/3

(C) Passes through (1, 2) (R) = 17/41


(D) || to x-axis

(S) = 3

EXERCISE # 3
Q.1

Q.2

Q.3

Q.10

A straight line through A (2, 3) cuts the lines


x + 3y = 9 and x + y + 1 = 0 at B and C
respectively. Find the equation of the line if
AB . AC = 20.

Q.11

Two sides of a rhombus, lying in the first quadrant,


are given by 3x 4y = 0 & 12x 5y = 0. If the
length of longer diagonal is 12, find the equations
of the other two sides of the rhombus

Q.12

From a point (2, 3) a ray of light is sent at an


angle (tan = 3) to the axis of x. Upon reaching
the x-axis the ray is reflected from it. Find the
equation of the straight line which contains the
reflected ray.

Q.13

Find the vertices of a ABC with A (1, 3) as a


vertex and x 2y + 1= 0, y 1 = 0 as the
equations of two of its medians.

Q.14

In a triangle ABC, coordinates of A are (1, 2) and


the equations to the medians through B and C are
x + y = 5 and x = 4 respectively. Find the
coordinates of B and C.

Two parallel straight line inclined at an angle of


135 to the axis of x meet the x-axis at the points
A, B and y-axis at the points C and D respectively.
Find the equation of the locus of point of
intersection of AD and BC.

Q.15

ABC is a variable triangle with a fixed centroid (5,


5). The side BC = 13 and B and C move on the x
and y axes respectively. Find the equation of the
locus of the vertex A.

The line 3x + 2y = 24 meets the y-axis at A and the


x-axis at B. The perpendicular bisector of AB
meets the line through (0, 1) parallel to
x-axis at C. Find the area of the triangle ABC.

Q.16

If the line

The coordinates of the points A, B and C are


respectively (6, 3), (3, 5) and (4, 2) and the
coordinates of P are (x, y), prove that
ar(PBC) : ar(ABC) = | x + y 2 | : 7.
The area of a triangle is 3/2 sq. units. Two of its
vertices are the points A (2, 3) and B (3, 2), the
centroid of the triangle lies on the line
3x y 8 = 0. Find the third vertex C.
The altitudes of a ABC are respectively AD, BE
and CF. If the points A, D, E and F have the

16 23
,
, (4, 1) and (1, 4)
5
5

coordinates (4, 5),

respectively, find the other vertices.


Q.4

Prove that the orthocentre of the triangle


whose angular points are (a cos r, a sin r);
r = 1, 2, 3 is the point (a cos r , a sin r).

Q.5

Q.6

Q.7

Q.8

foot of the perpendicular from the origin on the


straight line describes the circle x2 + y2 = c2.
Q.17

The equation of the altitude AD, BE, CF of a


triangle ABC are x + y = 0, x 4y = 0 and
2x y = 0 respectively. The co-ordinates of A are
(t, t). Find coordinates of B and C. Prove that if t
varies the locus of the centroid of the triangle ABC
is x + 5y = 0.

Q.18

If the straight lines ax + by + p = 0 and


x cos + y sin = p enclose an angle of
/4 between them and meet the line

Let a new distance d (P, Q) between the points P =


(x1, y1) and Q = (x2, y2) be defined as
d (P, Q) = |x 1 x2| + |y1 y2|. Let O = (0, 0) and A =
(3, 2) be two fixed points. Let R = (x, y),
x 0, y 0 such that R is equidistant from the
points O and A in the sense of the new distance.
Prove that the locus of R consists of a line segment
of finite length and an infinite ray.

Q.9

1
1
1
2 2 where c is a constant, prove that the
2
a
b
c

A line AB of length 2 moves with the end A


always on the x-axis and the end B on the line
y = 6x. Find the equation of the locus of the
middle point of AB.

Find the coordinates of the vertices of a square


inscribed in the triangle with vertices A(0,0),
B(2, 1), C(3, 0); given that two of its vertices are
on the side AC.

x y
1 moves in such a way that
a b

x sin = y cos in the same point, then show that


a2 + b2 = 2.
Q.19

Prove that all lines represented by the equation


(2cos+ 3sin) x + (3 cos 5 sin)y
(5 cos 2 sin) = 0 pass through a fixed point for

all values of . Find the coordinates of this point

Passage-II (Q. 29 to 31)


Let B1 3x + 4y 10 = 0 and B2 4x 3y 5 = 0

and its reflection in the line x + y = 2 .

are the bisectors of angle between lines L1 = 0 and L2


= 0. If L1 passes through origin and L2 = 0 meets the
curve y2 = 4ax at A and B and P is the point of
intersection of L1 = 0 and L2 = 0 then
On the basis of above passage, answer the
following questions :

Q.20 A variable line through the point (6/5, 6/5) cuts the
coordinate axes in the points A and B. If the point
P divides AB internally in the ratio 2 : 1, show that
the equation to the locus of P is
5xy = 2 (2x + y).
Q.21

Q.22

Q.23

A variable straight line passes through a fixed


point (h, k). Find the locus of the foot of the
perpendicular on it drawn from the origin.

Q.29

Find the equation of the two straight lines which


together with those given by the equation 6x 2 xy
y2 + x + 12y 35 = 0 will make a parallelogram
whose diagonals intersect in the origin.

Equation of line L2 = 0 is(A) 11x + 2y = 24


(B) 11x 2y = 20
(C) 5x + 3y = 13
(D) None of these

Q30

If AB subtends 90 angle at origin, then a is equal


to-

Prove that the straight lines joining the origin to


the points of intersection of the line 7x y + 2 = 0
and the curve 2x2 + y2 + x + y = 0 are at right
angles to one another.

Q.24

Find the equations of the pair of lines both of


which pass through the point (1, 2) and are parallel
to the bisectors of the angles between the lines
given by x2 + xy 2y2 + 4x y + 3 = 0.

Q.25

A variable line L passing through the point


B(2, 5) intersects the lines 2x2 5xy + 2y2 = 0 at P
and Q. Find the locus of the point R on L such that
distances BP, BR and BQ are in harmonic
progression.

Passage-I (Q. 26 to 28)


Let ABC be an acute angled triangle and AD, BE
and CF are its medians, where E and F are the
points E(3, 4) and F(1, 2) respectively and centroid
of ABC is G(3, 2), then
On the basis of above passage, answer the
following questions :
Q.26

Q.27

Q.28

The equation of side AB is (A) 2x + y = 4


(B) x + y 3 = 0
(C) 4x 2y = 0
(D) None of these
Co-ordinate of D are (A) (7, 4)
(B) (5, 0)
(C) (7, 4)
(D) (3, 0)

Q.31

(B) 3

(C) 6

(D) 2

4
11

(B)

3
11

(C)

5
11

(D) None of these

1
, then absolute value of product PA.PB is
4
equal toIf a =

(A)

2 2
11

(C)

3 24
5

(B)

125
121
(D) None of

these
Passage-III (Q. 32 to 34)
A (x1, y1), B(x2, y2), (y1 < y2) are two points on the
line x + y = 4 from which perpendicular AQ and
BP are drawn on line 4x + 3y = 10 where P and Q
are the feet of perpendiculars such that AQ = BP =
1. Now considering AB as diameter of a circle is
drawn
which
meets
the
line
4x + 3y = 10 at C and D such that C is closer to P.
On the basis of above passage, answer the
following questions :
Q.32

The value of

y1 y 2
is equal tox1 x 2

(A) 4
(C) 4
Q.33

(B) 3
(D) None of these

The length PQ is equal to(A) 3 14


(C)14

Height of altitude drawn from point A is


(in units) (A) 4 2

(A)

Q.34

(B) 4 5
(D) None of these

Length QD is equal to(A)

17

(B)

(C)

17

(D) 5

2 7

17

Passage-IV (Q. 35 to 37)


Let ABC be a triangle whose vertex A is (3, 4). L 1
= 0 and L2 = 0 are the angle bisectors of angles B
and C respectively where L1 x + 2y 5 = 0, L2 x
2y 3 = 0.
On the basis of above passage, answer the
following questions:
Q.35

Q.36

Slope of side BC is1


(A)
15
2
(C)
7

2
15

(D) None of these

Co-ordinate of point B is 73 24
21 29
,
,

(A)
(B)
5
5
5 10

110 203
,

5
3

(C)
Q.37

(B)

(D) None of these

Distance A is equal to (where I = Incentre) (A)

51
2

(B)

53
2

(C)

55
2

(D) None of these

EXERCISE # 4
Q.1

Q.2

Q.3

Q.4

Q.5

Q.6

Q.7

Let PS be the median of the triangle with vertices


P(2, 2), Q(6, 1) and R(7, 3). The equation of the
line passing through (1,1) and parallel to PS is[IIT-Screening-2000]
(A) 2x 9y 7 = 0
(B) 2x 9y 11 = 0
(C) 2x + 9y 11 = 0
(D) 2x + 9y + 7 = 0
Find the number of integer value of m which
makes the x coordinates of point of intersection of
lines 3x + 4y = 9 and y = mx + 1 integer.
[IIT-Screening-2001]
(A) 2
(B) 0
(C) 4
(D) 1
Area of the parallelogram formed by the lines
y = mx, y = mx + 1, y = nx, y = nx + 1 is[IIT-Screening-2001]
2
(A) |m + n| / (m n)
(B) 2 / |m + n|
(C) 1 / |m + n|
(D) 1 / |m n|

Through P and Q two straight lines L 1 and L2 are


drawn, parallel to 2x y = 5 and 3x + y = 5
respectively. Lines L1 and L2 intersect at R. Show
that the locus of R, as L varies, is a straight line.
[IIT-2002]
Q.8

No. of points with integer coordinates lie inside


the triangle whose vertices are (0, 0), (0, 21), (21,
0) is:
[IIT Screening 2003]
(A) 190
(B) 185
(C) 210
(D) 230

Q.9

Orthocentre of the triangle whose vertices are


A (0, 0), B (3, 4) & C (4, 0) is [IIT Scr. 2003]
3
5

(A) 3,
(B) 3,
4
4

(C) (3, 12)

(D) (2, 0)

Q.10

A pair of straight line x2 8x + 12 = 0 and


y2 14y + 45 = 0 are forming a square. What is the
centre of circle inscribed in the square
[IIT-Screening-2003]
(A) (3, 2) (B) (7, 4) (C) (4, 7) (D) (0, 1)

Q.11

Area of the triangle formed by the line x + y = 3 and


the angle bisector of the pair of lines
x2 y2 + 2y = 1 is
[IIT-Screening-2004]
(A) 1
(B) 3
(C) 2
(D) 4

Let P = (1, 0), Q = (0, 0) and R = (3, 3 3 ) be


three points. Then the equation of the bisector of
the angle PQR is [IIT-Screening-2002]
(A) ( 3 /2) x + y = 0 (B) x + 3 y = 0

Q.12

A line passes through the point P(h, k) is parallel


to the x- axis. It forms a triangle with the lines y =
x & x + y = 2 of area 4h 2 then find the locus of P.
[IIT 2005]

(C)

Q.13

If f(x) = min{1, x2, x3}, then


(A) f (x) > 0 x R

A straight line through the origin O meets the


parallel lines 4x + 2y = 9 and 2x + y + 6 = 0 at the
points P and Q respectively. Then the point O
divides the segment PQ in the ratio[IIT-Screening-2002]
(A) 1 : 2 (B) 3 : 4 (C) 2 : 1
(D) 4 : 3

3 x+y=0

(D) x + ( 3 /2) y = 0

Let 0 < < /2 be a fixed angle. If


P = (cos , sin ) and Q=(cos (), sin ())
then Q is obtained from P by[IIT-Screening-2002]
(A) clockwise rotation around origin through an
angle
(B) anticlockwise rotation around origin through an
angle
(C) reflection in the line through origin with slope
tan
(D) reflection in the line through origin with slope
tan /2
A straight line L through the origin meets the lines
x + y = 1 and x + y = 3 at P and Q respectively.

[IIT-2006]

(B) f(x) is continuous x R


(C) f(x) is not differentiable for two values of x
(D) f(x) is not differentiable but continuous
xR
Q.14

Q.15

Given an isosceles triangle, whose one angle is


120 and in-radius is 3 . So the area of triangle
is [IIT-2006]
(A) 4
(B) 12 + 7 3
(C) 7 + 12 3
(D) 12 7 3
Let O (0, 0), P(3, 4), Q (6, 0) be the vertices of the
triangle OPQ. The point R inside the triangle OPQ
is such that the triangles OPR, PQR, OQR are of

54

equal area. The coordinates of R are


[IIT-2007]

4
, 3
3

(A)

(B) 3,
3

(C) 3,
3

Q.16

4 2
,
3 3

(D)

Lines L1 : y x = 0 and L 2 : 2x + y = 0 intersect


the line L3 : y + 2 = 0 at P and Q, respectively. The
bisector of the acute angle between L 1 and L2
intersects L3 at R.
[IIT-2007]
Statement-1 : The ratio PR : RQ equals

Q.18

2 2 : 5
because
Statement-2 : In any triangle, bisector of an angle
divides the triangle into two similar triangles.
(A) Statement1 is True, Statement2 is True;
Statement2 is a correct explanation for
Statement1.
(B) Statement1 is True, Statement2 is True;
Statement2 is not a correct explanation for
Statement1
(C) Statement1 is True, Statement2 is False
(D) Statement1 is False, Statement2 is True
Q. 17

Consider three points P = ( sin ( ), cos ), Q


= (cos ( ), sin ) and R = (cos ( + ), sin

. Then 4
[IIT 2008]
(A) P lies on the line segment RQ
(B) Q lies on the line segment PR
(C) R lies on the line segment QP
(D) P, Q, R are non-collinear
( )), where 0 < , , <

Q.19

Consider the lines given by


L1 : x + 3y 5 = 0
L2: 3x ky 1= 0
L3 : 5x + 2y 12 = 0
Match the Statements/Expressions in Column-I
with the Statements/Expressions in Column-II
and indicate your answer by darkening
appropriate bubbles in the 4 4 matrix given in
the ORS.
[IIT 2008]
Column-I
Column-II
(A ) L1, L2, L3 are concurrent, if
(P) k = 9
6
(B ) One of L1, L2, L3 is parallel (Q) k =
5
to at least one of the other
two, if
5
(C ) L1, L2, L3 form a triangle, if (R) k =
6
(D) L1, L2, L3 do not form a
(S) k = 5
triangle, if
A straight line L through the point (3, 2) is
inclined at an angle 60 to the line 3 x + y = 1. If
L also intersects the x-axis, then the equation of L
is
[IIT-2011]
(A) y + 3 x + 2 3 3 = 0
(B) y

3 x+2+3

3 =0

(C)

3 yx+3+2

3 =0

(D)

3 y+x3+2

3 =0

54

EXERCISE # 5
Q.1

The straight line 5x + 4y = 0 passes through the point


of
intersection
of
the
straight
lines
x + 2y 10 = 0 and 2x + y + 5 = 0. [T/F]
[IIT 1983]

Q.2

Given the points A (0, 4) and B (0, 4), the


equation of the locus of the point P(x, y) such that
|AP BP| = 6 is..............
[IIT- 83]

Q.3

The end A, B of a straight line segment of constant


length c slide upon the fixed rectangular axes OX,
OY respectively. If the rectangle OAPB be
completed, then show that the locus of the foot of

Q.11

If P = (1, 0), Q = (1, 0) and R = (2, 0) are three


given points, then locus of the point S satisfying
the relation SQ2 + SR2 = 2SP2, is[IIT 1988]
(A) a straight line parallel to x-axis
(B) a circle passing through the origin
(C) a circle with the centre at the origin
(D) a straight line parallel to y-axis

Q.12

The lines 2x + 3y + 19 = 0 and 9x + 6y 17 = 0 cut


the coordinate axes in concyclic points. [T/F]
[IIT 1988]

Q.13

One of the diameter of the circle circumscribing the


rectangle ABCD is 4y = x + 7. If A and B are the
points (3, 4) and (5, 4) respectively, then find the
area of rectangle.
[IIT 1985]

Lines L1 ax + by + c = 0 and L 2 lx + my + n = 0
intersect at the point P and make an angle with
each other. Find the equation of a line L different
from L2 which passes through P and makes the
same angle with L1. [IIT 1988]

Q.14

Line L has intercepts a and b on the coordinate


axes. When the axes are rotated through a given
angle, keeping the origin fixed, the same line L has
intercepts p and q, then [IIT 1990]
(A) a2 + b2 = p2 + q2
1
1
1
1
(B) 2 + 2 = 2 + 2
q
p
a
b

the perpendicular drawn from P to AB is


2

y 3 = c3 .

Q.4

Q.5

Q.6

x3

[IIT 1983]

Three lines px + qy + r = 0, qx + ry + p = 0 and


rx + py + q = 0 are concurrent if [IIT 1985]
(A) p + q + r = 0
(B) p2 + q2 + r2 = pr + rp + pq
(C) p3 + q3 + r3 = 3 pqr
(D) None of these
The orthocentre of the triangle formed by the lines
x + y = 1, 2x + 3y = 6 and 4x y + 4 = 0 lies in the
quadrant number............ [IIT 1985]

(C) a2 + p2 = b2 + q2
1
1
1
1
(D) 2 + 2 = 2 + 2
p
q
a
b
Q.15

Straight lines 3x + 4y = 5 and 4x 3y = 15


intersect at the point A. Points B and C are chosen
on
these
two
lines
such
that
AB = AC. Determine the possible equations of the
line BC passing through the point (1, 2).
[IIT 1990]
A line cuts the x-axis at A(7, 0) and the
y-axis at B (0, 5). A variable line PQ is drawn
perpendicular to AB cutting the x-axis in P and the yaxis in Q. If AQ and BP intersect at R, find the locus
of R.
[IIT 1990]
Find the equation of the line passing through the
point (2, 3) and making intercept of length
2 units between the lines y + 2x = 3 and
y + 2x = 5.
[IIT 1991]

Q.7

Two sides of a rhombus ABCD are parallel to the


lines y = x + 2 and y = 7x + 3. If the diagonals of
the rhombus intersect at the point
(1, 2) and the vertex A is on the y- axis, find
possible coordinates of A.
[IIT 1985]

Q.8

The equations of the perpendicular bisectors of the


sides AB & AC of a triangle ABC are x y + 5 = 0
and x + 2y = 0, respectively. If the point A is
(1,2), find the equation of the line BC.[IIT 1986]

Q.16

The points 0, , (1, 3) and (82, 30) are


3

vertices of
[IIT 1986]
(A) an obtuse angled triangle
(B) an acute angled triangle
(C) a right angled triangle
(D) None of these
All points lying inside the triangle formed by the
points (1, 3), (5, 0) and (1, 2) satisfy
[IIT 1986]
(A) 3x + 2y 0
(B) 2x + y 13 0
(C) 2x 3y 12 0
(D) 2x + y 0

Q.17

Q.9

Q.10

Q.18

Show that all chords of the curve


3x2 y2 2x + 4y = 0, which subtend a right angle
at the origin, pass through a fixed point. Find the
coordinates of the point.
[IIT 1991]

Q.19

Let the algebraic sum of the perpendicular distance


from the points (2, 0), (0, 2) and
(1, 1) to a variable straight line be zero; then the

line passes through a fixed point whose


coordinates are ...................
[IIT 1991]
Q.20

Q.21

If the sum of the distances of a point from two


perpendicular lines in a plane is 1, then its locus is
[IIT 1992]
(A) square
(B) circle
(C) straight line
(D) two intersecting lines

The diagonals of parallelogram PQRS are along


the lines x + 3y = 4 and 6x 2y = 7. Then PQRS
must be a
[IIT-1998]
(A) rectangle
(B) square
(C) cyclic quadrilateral
(D) rhombus

Q.29

If the vertices P, Q, R of a triangle PQR are rational


points, which of the following points of the triangle
PQR is (are) always rational points (s) ?
[IIT-1998]
(A) Centroid
(B) Incentre
(C) Circumcentre
(D) Orthocentre

Q.30

If x1, x2, x3 as well as y1, y2, y3 are in G.P. with the


same common ratio, then the points (x 1, y1), (x2,
y2) and (x3, y3)
[IIT-1999]
(A) lie on a straight line
(B) lie on an ellipse
(C) lie on a circle
(D) are vertices of a triangle

Q.31

Let PQR be a right angled isosceles triangle, right


angled at P(2, 1). If the equation of the line QR is
2x + y = 3, then the equation representing the pair
of lines PQ and PR is[IIT-99]
2
2
(A) 3x 3y + 8xy + 20x + 10y + 25 = 0
(B) 3x2 3y2 + 8xy 20x 10y + 25 = 0
(C) 3x2 3y2 + 8xy + 10x + 15y + 20 = 0
(D) 3x2 3y2 8xy 10x 15y 20 = 0

Determine all values of for which the point


(2) lies inside the triangle formed by the lines
2x + 3y 1 = 0
x + 2y 3 = 0
5x 6y 1 = 0
[IIT 1992]

Q.22

A line through A (5, 4) meets the lines


x + 3y + 2 = 0, 2x + y + 4 = 0 and x y 5 = 0 at
the points B, C, and D respectively. If
(15/AB)2 + (10/AC)2 = (6/AD)2, find the equation
of the line.
[IIT 1993]

Q.23

The vertices of a triangle are A (1, 7),


B (5, 1) and C (1, 4). The equation of the bisector
of the ABC is........
[IIT 1993]

Q.24

The locus of a variable point whose distance from


(2, 0) is

2
times its distance from the line x =
3

9
is 2
(A) ellipse
(C) hyperbola

[IIT 1994]
(B) parabola
(D) None of these

Q.25

The equations to a pair of opposite sides of


parallelogram are x2 5x + 6 = 0 and
y2 6y + 5 = 0, the equations to its diagonals are
[IIT 1994]
(A) x + 4y = 13, y = 4x 7
(B) 4x + y = 13, 4y = x 7
(C) 4x + y = 13, y = 4x 7
(D) y 4x = 13, y + 4x = 7

Q.26

The orthocentre of the triangle formed by the lines


xy = 0 and x + y = 1 is
[IIT 1995]
1 1
1 1
, (B)
, (C) (0, 0)
(A)
(D)
2
2
3 3

1 1
,

4 4
Q.27

Q.28

A rectangle PQRS has its side PQ parallel to the


line y = mx and vertices P, Q and S on the lines y
= a , x = b and x = b, respectively. Find the
locus of the vertex R.
[IIT - 96]

ANSWER KEY
EXERCISE # 1

45. False

46. False

47. False

52. (5/2, 1/4)

53. 2

54. 2

48. 1

49. 7

50. (p, q)

51. 2

EXERCISE # 2
(PART-A)

(PART-B)

(PART-C)

(PART-D)
48. A S; B P ; C Q; D R

49. A P; B Q ; C R; D S

50 A R, S; B P, Q; C S; D Q

51 A R; B P ; C S; D Q

52 A S; B R ; C P; D Q

EXERCISE # 3
2. (1, 1) or (2, 10)

3. (0, 7), (8, 1)

5. (4, 3), (7, 2)

6. x2 + y2 30x 30y + 281 = 0

7. 9x2 + 10y2 6xy 92= 0

9.

10. x y = 1, 3x y + 3 = 0

11. 3x 4y +

13. B(5, 1), C( 3, 4)

14. x y = 0

t
t
2
, t , B
t,

6
2
3

180
130

3 3
,
2 4

= 0; 12x 5y =

468
130

19. (1, 1), (

22. 6x2 xy y2 x 12y 35 = 0

23. x + 2y 7 = 0, x 4y = 1, x y + 2 = 0

24. x2 6xy y2 + 10x + 10y 15 = 0

25. 17x 10y = 0

30. (C)

31. (B)

12. 3x + y + 9 = 0

15. 91

17. C

29. (B)

3 9 3 9
,0 ,
,
,0

2 4 4 4

32. (B)

1,

1)

21. x2 + y2 hx ky = 0

26. (A)

27. (B)

28. (C)

33. (C)

34. (D)

35. (C)

36. (A)

37. (B)

EXERCISE # 4
1. (D)

2. (A)

3. (D)

4. (B)

5. (C)

6. (D)

7. x 3y + 5 = 0

8. (A)

9. (A)

10. (C)

11. (C)

12. 4x2 = (y 1)2

14. (B)

15. (C)

16. (C)

17. (D)

18. A S; B P,Q; C R; D P,Q,S

13. (B,D)

19. (B)

EXERCISE # 5
1. True

2.

y2
x2

=1
9
7

8. 14x + 23y 40 = 0

9. (D)

4. 32 sq. units

10. (A,C)

13. (a2 + b2) (lx + my + n) 2 (al + bm) (ax + by + c) = 0


2

16. x + y 7x + 5y = 0

21. ,1
2

26. (C)

1
,1
2

5. (A,B,C)

6. 1st quadrant

7. (0, 0) or (0, 5/2)

11. (D)

12. True

14. (B)

15. x 7y +13 = 0 or 7x + y 9 = 0

17. 3x + 4y 18 = 0 or x 2 = 0 18. (1, 2)

19. (1, 1)

20. (A)

22. 2x + 3y + 22 = 0

24. (A)

25. (C)

30. (A)

31. (B)

23. x 7y + 2 = 0

27. (m2 1) x my + b (m2 + 1) + am = 0 28. (D) 29. (A,C,D)

You might also like